OB Test 3 (Complicated Newborn & Complicated Pregnancy)

अब Quizwiz के साथ अपने होमवर्क और परीक्षाओं को एस करें!

The nurse is collecting data from a pregnant pt in the second trimester of pregnancy who was admitted to the maternity unit with a suspected diagnosis of abruptio placentae. Which findings are associated with abruptio placentae? Select all that apply. A. Uterine tenderness B. Acute abdominal pain C. A hard, "board-like" abdomen D. Painless, bright red vaginal bleeding E. Increased uterine resting tone on fetal monitoring

A, B, C, E (not D bc it would be painful dark red bleeding)

22. Which order should the nurse expect for a client admitted with a threatened abortion? a. Bed rest b. Administration of ritodrine IV c. Nothing by mouth (nil per os [NPO]) d. Narcotic analgesia every 3 hours, as needed

ANS: A Decreasing the woman's activity level may alleviate the bleeding and allow the pregnancy to continue. Ritodrine is not the first drug of choice for tocolytic medications. Having the woman placed on NPO is unnecessary. At times, dehydration may produce contractions; therefore, hydration is important. Narcotic analgesia will not decrease the contractions and may mask the severity of the contractions.

The nurse reviews the assessment hx for a pt w a suspected ectopic pregnancy. Which assessment findings predispose the pt to an ectopic pregnancy? SATA. A. Use of diaphragm B. Use of fertility medications C. History of Chlamydia D. Use of an intrauterine device E. History of pelvic inflammatory disease (PID)

B, C, D, E

The nurse is performing a prenatal assessment on a pregnant pt. The nurse should plan to implement teaching related to risk for abruptio placentae if which information is obtained on assessment? A. The client is 28 years of age. B. This is the second pregnancy. C. The client has a history of hypertension. D. The client performs moderate exercise on a regular daily schedule.

C

A newborn's birth was prolonged because the fetal shoulders were very wide. Which reflex would the nurse anticipate a problem with? a. Stepping b. Plantar c. Babinski d. Moro

d. Moro A difficult birth due to broad fetal shoulders may result in a fractured clavicle, as evidenced by a knot or lump, limited arm movement, and a unilateral Moro reflex. Plantar reflex is unrelated to a difficult birth caused by a fetus with broad shoulders. Babinski reflex involves the feet; it is not related to a difficult birth caused by a fetus with broad shoulders. Stepping reflex involves the feet; it is not related to a difficult birth caused by a fetus with broad shoulders.

A baby has just been born to a type 1 diabetic mother with retinopathy and nephropathy. Which of the following neonatal findings would the nurse expect to see? 1. Hyperalbuminemia. 2. Polycythemia. 3. Hypercalcemia. 4. Hypoinsulinemia.

2 Because the placenta is likely to be functioning less than optimally, it is highly likely that the baby will be polycythemic. The increase in red blood cells would improve the baby's oxygenation in utero.

16. In contrast to placenta previa, what is the most prevalent clinical manifestation of abruptio placentae? a. Bleeding b. Intense abdominal pain c. Uterine activity d. Cramping

ANS: B Pain is absent with placenta previa and may be agonizing with abruptio placentae. Bleeding may be present in varying degrees for both placental conditions. Uterine activity and cramping may be present with both placental conditions.

3. The nurse is preparing to administer methotrexate to the client. This hazardous drug is most often used for which obstetric complication? a. Complete hydatidiform mole b. Missed abortion c. Unruptured ectopic pregnancy d. Abruptio placentae

ANS: C Methotrexate is an effective nonsurgical treatment option for a hemodynamically stable woman whose ectopic pregnancy is unruptured and measures less than 4 cm in diameter. Methotrexate is not indicated or recommended as a treatment option for a complete hydatidiform mole, for a missed abortion, or for abruptio placentae.

A client with preeclampsia is admitted complaining of pounding headache, visual changes, and epigastric pain. Nursing care is based on the knowledge that these signs indicate: a. gastrointestinal upset. b. effects of magnesium sulfate. c. anxiety caused by hospitalization. d. worsening disease and impending convulsion.

D ~ Headache and visual disturbances are caused by increased cerebral edema. Epigastric pain indicates distention of the hepatic capsules and often warns that a convulsion is imminent. Gastrointestinal upset is not an indication as severe as the headache and visual disturbance. She has not yet been started on magnesium sulfate as a treatment. The signs and symptoms do not describe anxiety.

A primigravida of 28 years of age is admitted to the antepartum unit with a diagnosis of hyperemesis gravidarum. Nursing care is based on which of the following? a. She should be isolated from her family. b. This condition is caused by psychogenic factors. c. The treatment is similar to that for morning sickness. d. She should be assessed for signs of dehydration and starvation.

D ~ The cause of hyperemesis gravidarum is unknown, but dehydration and starvation are the major complications. Emotional support is essential to the care of this client. She needs the opportunity to express how it feels to live with constant nausea. The cause is unknown. The first attempts to control the nausea are to treat it like morning sickness, but if treatment is not successful, further care is needed.

The nurse is admitting a neonate two hours after delivery. About which assessment data should the nurse be concerned?Select all that apply. a. Nasal flaring b. Hands and feet are blue c. Apical heart rate 156 d. Minimal response to verbal stimulation e. Retractions

a, e Explanation: Distal cyanosis This is a normal finding for a neonate at 2 hours of age. Nasal flaring could be a sign of respiratory distress, and requires immediate intervention. Retractions could be a sign of respiratory distress, and requires immediate intervention. Minimal response to verball stimuli and a apical heart rate are normal for 2 hours after delivery

The nurse is developing a plan of care for an infant born at 28 weeks' gestation. What would be a realistic goal for this infant to achieve within one week? a. Maintaining respiratory rate at 30-60 breaths/minute b. Maintaining body temperature in a bassinet c. Recognizing the parents d. Drinking from a bottle

a. Maintaining respiratory rate at 30-60 breaths/minute Explanation: Drinking from a bottle is not a timely goal for a 28-week-gestation infant at 1 week of age. Recognizing parents is not a timely goal for a 28-weeks-gestation infant at 1 week of age. A healthy respiratory rate for all newborns is 30-60 breaths/min. Maintaining body temperature in a bassinet is not a timely goal for a 28-week-gestation infant at 1 week of age.

An infant of a diabetic mother (IDM) is admitted to the newborn nursery. Which nursing intervention has highest priority at this time? a. Complete a gestational age assessment. b. Assess the infant's blood glucose level. c. Administer vitamin K intramuscularly. d. Clean the umbilical cord.

b. Assess the infant's blood glucose level. Explanation: Cleaning the umbilical cord is important, but is not the highest priority for a newborn of a diabetic mother. Administering vitamin K is important, but is not the highest priority for a newborn of a diabetic mother. Completing a gestational age assessment is important, but is not the highest priority for a newborn of a diabetic mother. An infant of a diabetic mother is at risk for hypoglycemia, and blood glucose should be monitored closely after delivery and treated if necessary.

A 38 week gestation neonate born at 5 lb 3 oz was noted to not pass the hearing test and has a small head circumference. Which of the following do you expect this newborn to be experiencing? a. herpes simplex virus b. cytomegalovirus c. toxoplasmosis d. rubella

b. cytomegalovirus

Four full-term babies were admitted to the neonatal nursery. The mothers of each of the babies had labors of 4 hours or less. The nursery nurse should carefully monitor which of the babies for hypothermia? 1. The baby whose mother cultured positive for group B strep during her third trimester. 2. The baby whose mother had gestational diabetes. 3. The baby whose mother was hospitalized for 3 months with complete placenta previa. 4. The baby whose mother previously had a stillbirth.

1 Group B streptococcus causes severe infections in the newborn. A sign of neonatal sepsis is hypothermia. (2. Babies whose mothers had gestational diabetes (GDM) should be carefully monitored for hypoglycemia rather than for hypothermia. 3. There is no relationship between placenta previa and neonatal hypothermia. 4. There is no evidence from the question that the stillbirth was related to a gestational infection.)

A nurse is preparing to care for a newborn who has respiratory distress syndrome. Which initial action should the nurse plan to best facilitate bonding between the newborn and the parents? 1. Encourage the parents to touch their newborn. 2. Identify specific caregiving tasks that may be assumed by the parents. 3. Explain the equipment that is used and how it functions to assist their newborn. 4. Give the parents pamphlets that will help them understand their newborn's condition.

1 The best initial action to begin the attachment process and promote bonding is to encourage the parents to touch their newborn. The parents' initial need is to become acquainted with their newborn. Option 2 may be frightening to the parents because of the condition of the newborn and the unfamiliarity of high-risk newborn care practices. This option will be appropriate later, as the newborn's condition becomes stable. Option 3 is important but is not specific to parent-newborn bonding activities. Option 4 is inappropriate initially. Requiring parents to focus on pamphlets or literature does not enhance the parent-newborn bond.

Four 38-week-gestation gravidas have just delivered. Which of the babies should be monitored closely by the nurse for respiratory distress? 1. The baby whose mother has diabetes mellitus. 2. The baby whose mother has lung cancer. 3. The baby whose mother has hypothyroidism. 4. The baby whose mother has asthma.

1 The lung maturation of infants of diabetic mothers is often delayed. These babies must be monitored at birth for respiratory distress.

The nurse develops a plan of care for a woman with human immunodeficiency virus infection and her newborn. The nurse should include which intervention in the plan of care? 1. Monitoring the newborn's vital signs routinely 2. Maintaining standard precautions at all times while caring for the newborn 3. Initiating referral to evaluate for blindness, deafness, learning problems, or behavioral problems 4. Instructing the breast-feeding mother regarding the treatment of the nipples with nystatin ointment

2 An infant born to a mother infected with human immunodeficiency virus (HIV) must be cared for with strict attention to standard precautions. This prevents the transmission of HIV from the newborn, if infected, to others and prevents transmission of other infectious agents to the possibly immunocompromised newborn. Mothers infected with HIV should not breast-feed. Options 1 and 3 are not associated specifically with the care of a potentially HIV-infected newborn.

A newborn nursery nurse notes that a 36-hour-old baby's body is jaundiced. Which of the following nursing interventions will be most therapeutic? 1. Maintain a warm ambient environment. 2. Have the mother feed the baby frequently. 3. Have the mother hold the baby skin to skin. 4. Place the baby naked by a closed sunlit window.

2 Bilirubin is excreted through the bowel. The more the baby consumes, the more stools, and therefore the more bilirubin the baby will expel.

A client who is positive for human immunodeficiency virus (HIV) delivers a newborn infant. The nurse provides instructions to help the client regarding care of her infant. Which client statement indicates the need for further instruction? 1. "I will be sure to wash my hands before and after bathroom use." 2. "I need to breast-feed, especially for the first 6 weeks postpartum." 3. "Support groups are available to assist me with understanding my diagnosis of HIV." 4. "My newborn infant should be on antiviral medications for the first 6 weeks after delivery."

2 The mode of perinatal transmission of human immunodeficiency virus (HIV) to the fetus or neonate of an HIV-positive woman can occur during the antenatal, intrapartal, or postpartum period. HIV transmission can occur during breast-feeding. HIV-positive clients should be encouraged to bottle-feed their infants per the health care provider's prescription. Frequent hand-washing is encouraged. Support groups and community agencies can be identified to assist the parents with the newborn infant's home care, the impact of the diagnosis of HIV infection, and available financial resources. It is recommended that infants of HIV-positive clients receive antiviral medications for the first 6 weeks of life.

The nurse weighing a term newborn during the initial newborn assessment determines the infant's weight to be 4325 g. The nurse determines that this infant may be at risk for which complications? Select all that apply. 1. Retinopathy 2. Hypoglycemia 3. Fractured clavicle 4. Hyperbilirubinemia 5. Congenital heart defect 6. Necrotizing enterocolitis

2, 3, 5 Any newborn weighing more than 4000 g at birth is defined as being large for gestational age (LGA). Because of their size, LGA infants are also at risk for hypoglycemia. LGA infants also have a higher incidence of birth injuries (fractured clavicle), asphyxia, and congenital anomalies (heart defect). Retinopathy is a disorder that affects the developing vessels of preterm infants. Hyperbilirubinemia is not an immediate risk related to LGA. Preterm birth is the most prominent risk factor in the development of necrotizing enterocolitis.

A nurse is monitoring a newborn infant who has been circumcised. The nurse notes that the infant has a temperature of 100.6° F and that the dressing at the circumcised area is saturated with a foul-smelling drainage. Which is the priority nursing action? 1. Reinforce the dressing. 2. Document the findings. 3. Contact the health care provider. 4. Swab the drainage and send the sample to the laboratory for culture.

3 Complications after circumcision include bleeding, failure to urinate, displacement of the Plastibell, and infection (indicated by a fever and a purulent or foul-smelling drainage). If signs of infection occur, the health care provider is notified. The nurse would change, not reinforce, the dressing; reinforcing the dressing leaves the foul smelling drainage in contact with the surgical site. The nurse would document the findings, but this is not the priority item. The health care provider will prescribe a culture if it is necessary; it is not within the realm of nursing responsibilities to prescribe a diagnostic test.

A baby has been admitted to the neonatal nursery whose mother is hepatitis B-surface antigen positive. Which of the following actions by the nurse should be taken at this time? 1. Monitor the baby for signs of hepatitis B. 2. Place the baby on contact isolation. 3. Obtain an order for the hepatitis B vaccine and the immune globulin. 4. Advise the mother that breastfeeding is absolutely contraindicated.

3. Babies exposed to hepatitis B in utero should receive the first dose of hepatitis B vaccine as well as hepatitis B immune globulin (HBIG) within 12 hours of delivery to reduce transmission of the virus. (1. Vertical transmission of hepatitis B does occur. Symptoms of the disease would not be evident during the neonatal period, however. 2. Standard precautions are sufficient for the care of the baby exposed to hepatitis B in utero. 4. Breastfeeding is not contraindicated when the mother is hepatitis B positive.)

A neonate is in the warming crib for poor thermoregulation. Which of the following sites is appropriate for the placement of the skin thermal sensor? 1. Xiphoid process. 2. Forehead. 3. Abdominal wall. 4. Great toe.

3. Abdominal wall.

An infant is born to a mother with hepatitis B. Which prophylactic measure would be indicated for the infant? 1. Hepatitis B vaccine given within 24 hours after birth 2. Immune globulin (IG) given as soon as possible after delivery 3. Hepatitis B immune globulin (HBIG) given within 14 days after birth 4. Hepatitis B immune globulin (HBIG) and hepatitis B vaccine given within 12 hours after birth

4 Both HBIG and the vaccine are given to infants with perinatal exposure to prevent hepatitis and achieve lifelong prophylaxis; they are administered within 12 hours after birth. IG is given to prevent hepatitis A.

The nurse is preparing to care for a newborn receiving phototherapy. Which interventions should be included in the plan of care? Select all that apply. 1. Avoid stimulation. 2. Decrease fluid intake. 3. Expose all of the newborn's skin. 4. Monitor skin temperature closely. 5. Reposition the newborn every 2 hours. 6. Cover the newborn's eyes with eye shields or patches.

4. Monitor skin temperature closely. 5. Reposition the newborn every 2 hours. 6. Cover the newborn's eyes with eye shields or patches. Phototherapy is the use of intense fluorescent lights to reduce serum bilirubin levels in the newborn. Adverse effects from treatment, such as eye damage, dehydration, or sensory deprivation, can occur. Interventions include exposing as much of the newborn's skin as possible; however, the genital area is covered. The newborn's eyes are also covered with eye shields or patches, ensuring that the eyelids are closed when shields or patches are applied. The shields or patches are removed at least once per shift to inspect the eyes for infection or irritation and to allow eye contact. The nurse measures the lamp energy output to ensure efficacy of the treatment (done with a special device known as a photometer), monitors skin temperature closely, and increases fluids to compensate for water loss. The newborn will have loose green stools and green-colored urine. The newborn's skin color is monitored with the fluorescent light turned off every 4 to 8 hours and is monitored for bronze baby syndrome, a grayish brown discoloration of the skin. The newborn is repositioned every 2 hours, and stimulation is provided. After treatment, the newborn is monitored for signs of hyperbilirubinemia because rebound elevations can occur after therapy is discontinued.

Which finding should be the nurse's priority in a client suspected as having gestational trophoblastic disease? a. Uterine contractions b. Nausea and vomiting c. Blood pressure of 130/80 mm Hg d. Increase discharge of vaginal mucus

A ~ Uterine contractions can cause trophoblastic tissue to be pulled into large venous sinusoids in the uterus, resulting in embolization of the tissue and respiratory distress. Nausea and vomiting and blood pressure of 130/80 mm Hg represent no immediate danger to the client and can be addressed later. Increased discharge of vaginal mucus is a normal finding in pregnancy.

Which routine nursing assessment is contraindicated for a client admitted with suspected placenta previa? a. Determining cervical dilation and effacement b. Monitoring FHR and maternal vital signs c. Observing vaginal bleeding or leakage of amniotic fluid d. Determining frequency, duration, and intensity of contractions

A ~ Vaginal examination of the cervix may result in perforation of the placenta and subsequent hemorrhage. Monitoring FHR and maternal vital signs is a necessary part of the assessment for this client. Monitoring for bleeding and rupture of membranes is not contraindicated with this client. Monitoring contractions is not contraindicated with this client.

The nurse who suspects that a client has early signs of ectopic pregnancy should be observing her for which symptoms? (SATA) a. Pelvic pain b. Missed period c. Abdominal pain d. Unanticipated heavy bleeding e. Vaginal spotting or light bleeding

A, B, C, E ~ A missed period or spotting can easily be mistaken by the client as early signs of pregnancy. More subtle signs depend on exactly where the implantation occurs. The nurse must be thorough in her assessment because pain is not a normal symptom of early pregnancy. As the fallopian tube tears open and the embryo is expelled, the client often exhibits severe pain accompanied by intraabdominal hemorrhage. This may progress to hypovolemic shock with minimal or even no external bleeding. In about 50% of women, shoulder and neck pain occurs because of irritation of the diaphragm from the hemorrhage.

The prenatal nurse is completing an assessment of a pregnant client at 36 weeks of gestation who has preeclampsia. Which question is important for the nurse to ask during the​ assessment? (Select all that​ apply.) A. "Have you noticed any changes in your​ vision?" B. "Have you had any episodes of​ diarrhea?" C. "Have you been having any nausea or​ vomiting?" D. "Have you had any​ headaches?" E. "Have you experienced any​ seizures?"

A,C,D,E ​Rationale: During the client​ interview, the nurse should ask the client about the presence of preeclampsia​ complications, including​ headache, changes in​ vision, presence of nausea or​ vomiting, dizziness, and seizures. Diarrhea is not characteristic of​ preeclampsia, so this question would not be necessary to ask.

18. What is the correct definition of a spontaneous termination of a pregnancy (abortion)? a. Pregnancy is less than 20 weeks. b. Fetus weighs less than 1000 g. c. Products of conception are passed intact. d. No evidence exists of intrauterine infection.

ANS: A An abortion is the termination of pregnancy before the age of viability (20 weeks). The weight of the fetus is not considered because some older fetuses may have a low birth weight. A spontaneous abortion may be complete or incomplete and may be caused by many problems, one being intrauterine infection.

21. What is the highest priority nursing intervention when admitting a pregnant woman who has experienced a bleeding episode in late pregnancy? a. Assessing FHR and maternal vital signs b. Performing a venipuncture for hemoglobin and hematocrit levels c. Placing clean disposable pads to collect any drainage d. Monitoring uterine contractions

ANS: A Assessment of the FHR and maternal vital signs will assist the nurse in determining the degree of the blood loss and its effect on the mother and fetus. The most important assessment is to check the well-being of both the mother and the fetus. The blood levels can be obtained later. Assessing future bleeding is important; however, the top priority remains mother/fetal well-being. Monitoring uterine contractions is important but not a top priority.

A pregnant woman was admitted for induction of labor at 43 weeks of gestation with sure dates. A nonstress test (NST) in the obstetrician's office revealed a nonreactive tracing. On artificial rupture of membranes, thick meconium-stained fluid was noted. What should the nurse caring for the infant after birth anticipate? a.Meconium aspiration, hypoglycemia, and dry, cracked skin b.Excessive vernix caseosa covering the skin, lethargy, and RDS c.Golden yellow to green-stained skin and nails, absence of scalp hair, and an increased amount of subcutaneous fat d.Hyperglycemia, hyperthermia, and an alert, wide-eyed appearance

ANS: A Meconium aspiration, hypoglycemia, and dry, cracked skin are consistent with a postmature infant. Excessive vernix caseosa, lethargy, and RDS are consistent with a very premature infant. The skin may be meconium stained, but the infant will most likely have long hair and decreased amounts of subcutaneous fat. Postmaturity with a nonreactive NST is indicative of hypoxia. Signs and symptoms associated with fetal hypoxia are hypoglycemia, temperature instability, and lethargy.

Screening at 24 weeks of gestation reveals that a pregnant woman has gestational diabetes mellitus (GDM). In planning her care the nurse and the woman mutually agree that an expected outcome is to prevent injury to the fetus as a result of GDM. The nurse identifies that the fetus is at greatest risk for: a. Macrosomia b. Congenital anomalies of the central nervous system c. Preterm birth d. Low birth weight

ANS: A Poor glycemic control later in pregnancy increases the rate of fetal macrosomia. Poor glycemic control during the preconception time frame and into the early weeks of the pregnancy is associated with congenital anomalies. Preterm labor or birth is more likely to occur with severe diabetes and is the greatest risk in women with pregestational diabetes. Increased weight, or macrosomia, is the greatest risk factor for this woman.

13. In caring for an immediate postpartum client, the nurse notes petechiae and oozing from her intravenous (IV) site. The client would be closely monitored for which clotting disorder? a. DIC b. Amniotic fluid embolism (AFE) c. Hemorrhage d. HELLP syndrome

ANS: A The diagnosis of DIC is made according to clinical findings and laboratory markers. A physical examination reveals unusual bleeding. Petechiae may appear around a blood pressure cuff on the woman's arm. Excessive bleeding may occur from the site of slight trauma such as venipuncture sites. These symptoms are not associated with AFE, nor is AFE a bleeding disorder. Hemorrhage occurs for a variety of reasons in the postpartum client. These symptoms are associated with DIC. Hemorrhage would be a finding associated with DIC and is not a clotting disorder in and of itself. HELLP syndrome is not a clotting disorder, but it may contribute to the clotting disorder DIC.

Infants born between 34 0/7 and 36 6/7 weeks of gestation are called late-preterm infants because they have many needs similar to those of preterm infants. Because they are more stable than early-preterm infants, they may receive care that is similar to that of a full-term baby. These infants are at increased risk for which conditions? (Select all that apply.) a.Problems with thermoregulation b.Cardiac distress c.Hyperbilirubinemia d.Sepsis e.Hyperglycemia

ANS: A, C, D Thermoregulation problems, hyperbilirubinemia, and sepsis are all conditions related to immaturity and warrant close observation. After discharge, the infant is at risk for rehospitalization related to these problems. Association of Women's Health, Obstetric and Neonatal Nurses (AWHONN) launched the Near-Term Infant Initiative to study the problem and ways to ensure that these infants receive adequate care. The nurse should ensure that this infant is adequately feeding before discharge and that parents are taught the signs and symptoms of these complications. Late-preterm infants are also at increased risk for respiratory distress and hypoglycemia.

1. A client who has undergone a D&C for early pregnancy loss is likely to be discharged the same day. The nurse must ensure that her vital signs are stable, that bleeding has been controlled, and that the woman has adequately recovered from the administration of anesthesia. To promote an optimal recovery, what information should discharge teaching include? (Select all that apply.) a. Iron supplementation b. Resumption of intercourse at 6 weeks postprocedure c. Referral to a support group, if necessary d. Expectation of heavy bleeding for at least 2 weeks e. Emphasizing the need for rest

ANS: A, C, E The woman should be advised to consume a diet high in iron and protein. For many women, iron supplementation also is necessary. The nurse should acknowledge that the client has experienced a loss, however early. She can be taught to expect mood swings and possibly depression. Referral to a support group, clergy, or professional counseling may be necessary. Discharge teaching should emphasize the need for rest. Nothing should be placed in the vagina for 2 weeks after the procedure, including tampons and vaginal intercourse. The purpose of this recommendation is to prevent infection. Should infection occur, antibiotics may be prescribed. The client should expect a scant, dark discharge for 1 to 2 weeks. Should heavy, profuse, or bright bleeding occur, she should be instructed to contact her health care provider.

NEC is an acute inflammatory disease of the gastrointestinal mucosa that can progress to perforation of the bowel. Approximately 2% to 5% of premature infants succumb to this fatal disease. Care is supportive; however, known interventions may decrease the risk of NEC. Which intervention has the greatest effect on lowering the risk of NEC? a.Early enteral feedings b.Breastfeeding c.Exchange transfusion d.Prophylactic probiotics

ANS: B A decrease in the incidence of NEC is directly correlated with exclusive breastfeeding. Breast milk enhances the maturation of the gastrointestinal tract and contains immune factors that contribute to a lower incidence or severity of NEC, Crohn disease, and celiac illness. The NICU nurse can be very supportive of the mother in terms of providing her with equipment to pump breast milk, ensuring privacy, and encouraging skin-to-skin contact with the infant. Early enteral feedings of formula or hyperosmolar feedings are a risk factor known to contribute to the development of NEC. The mother should be encouraged to pump or feed breast milk exclusively. Exchange transfusion may be necessary; however, it is a known risk factor for the development of NEC. Although still early, a study in 2005 found that the introduction of prophylactic probiotics appeared to enhance the normal flora of the bowel and therefore decrease the severity of NEC when it did occur. This treatment modality is not as widespread as encouraging breastfeeding; however, it is another strategy that the care providers of these extremely fragile infants may have at their disposal.

2. A perinatal nurse is giving discharge instructions to a woman, status postsuction, and curettage secondary to a hydatidiform mole. The woman asks why she must take oral contraceptives for the next 12 months. What is the bestresponse by the nurse? a. "If you get pregnant within 1 year, the chance of a successful pregnancy is very small. Therefore, if you desire a future pregnancy, it would be better for you to use the most reliable method of contraception available." b. "The major risk to you after a molar pregnancy is a type of cancer that can be diagnosed only by measuring the same hormone that your body produces during pregnancy. If you were to get pregnant, then it would make the diagnosis of this cancer more difficult." c. "If you can avoid a pregnancy for the next year, the chance of developing a second molar pregnancy is rare. Therefore, to improve your chance of a successful pregnancy, not getting pregnant at this time is best." d. "Oral contraceptives are the only form of birth control that will prevent a recurrence of a molar pregnancy."

ANS: B Beta-human chorionic gonadotropin (beta-hCG) hormone levels are drawn for 1 year to ensure that the mole is completely gone. The chance of developing choriocarcinoma after the development of a hydatidiform mole is increased. Therefore, the goal is to achieve a zero human chorionic gonadotropin (hCG) level. If the woman were to become pregnant, then it may obscure the presence of the potentially carcinogenic cells. Women should be instructed to use birth control for 1 year after treatment for a hydatidiform mole. The rationale for avoiding pregnancy for 1 year is to ensure that carcinogenic cells are not present. Any contraceptive method except an intrauterine device (IUD) is acceptable.

17. Which maternal condition always necessitates delivery by cesarean birth? a. Marginal placenta previa b. Complete placenta previa c. Ectopic pregnancy d. Eclampsia

ANS: B In complete placenta previa, the placenta completely covers the cervical os. A cesarean birth is the acceptable method of delivery. The risk of fetal death occurring is due to preterm birth. If the previa is marginal (i.e., 2 cm or greater away from the cervical os), then labor can be attempted. A cesarean birth is not indicated for an ectopic pregnancy. Labor can be safely induced if the eclampsia is under control.

A nurse caring for a woman hospitalized for hyperemesis gravidarum expects that initial treatment will involve: a. Corticosteroids to reduce inflammation b. IV therapy to correct fluid and electrolyte imbalances c. An antiemetic, such as pyridoxine, to control nausea and vomiting d. Enteral nutrition to correct nutritional deficits

ANS: B Initially, the woman who is unable to down clear liquids by mouth requires IV therapy for correction of fluid and electrolyte imbalances. Corticosteroids have been used successfully to treat refractory hyperemesis gravidarum, but they are not the expected initial treatment for this disorder. Pyridoxine is vitamin B6, not an antiemetic. Promethazine, a common antiemetic, may be prescribed. In severe cases of hyperemesis gravidarum, enteral nutrition via a feeding tube may be necessary to correct maternal nutritional deprivation. This is not the initial treatment for this client.

Preconception counseling is critical to the outcome of diabetic pregnancies because poor glycemic control before and during early pregnancy is associated with: a. Frequent episodes of maternal hypoglycemia b. Congenital anomalies in the fetus c. Polyhydramnios d. Hyperemesis gravidarum

ANS: B Preconception counseling is particularly important because strict metabolic control before conception and in the early weeks of gestation is instrumental in decreasing the risks of congenital anomalies. Frequent episodes of maternal hypoglycemia may occur during the first trimester (not before conception) as a result of hormone changes and the effects on insulin production and usage. Hydramnios occurs about 10 times more often in diabetic pregnancies than in nondiabetic pregnancies. Typically it is seen in the third trimester of pregnancy. Hyperemesis gravidarum may exacerbate hypoglycemic events because the decreased food intake by the mother and glucose transfer to the fetus contribute to hypoglycemia.

Necrotizing enterocolitis (NEC) is an inflammatory disease of the gastrointestinal mucosa. The signs of NEC are nonspecific. What are generalized signs and symptoms of this condition? a.Hypertonia, tachycardia, and metabolic alkalosis b.Abdominal distention, temperature instability, and grossly bloody stools c.Hypertension, absence of apnea, and ruddy skin color d.Scaphoid abdomen, no residual with feedings, and increased urinary output

ANS: B Some generalized signs of NEC include decreased activity, hypotonia, pallor, recurrent apnea and bradycardia, decreased oxygen saturation values, respiratory distress, metabolic acidosis, oliguria, hypotension, decreased perfusion, temperature instability, cyanosis, abdominal distention, residual gastric aspirates, vomiting, grossly bloody stools, abdominal tenderness, and erythema of the abdominal wall. The infant may display hypotonia, bradycardia, and metabolic acidosis.

4. A 26-year-old pregnant woman, gravida 2, para 1-0-0-1, is 28 weeks pregnant when she experiences bright red, painless vaginal bleeding. On her arrival at the hospital, which diagnostic procedure will the client most likely have performed? a. Amniocentesis for fetal lung maturity b. Transvaginal ultrasound for placental location c. Contraction stress test (CST) d. Internal fetal monitoring

ANS: B The presence of painless bleeding should always alert the health care team to the possibility of placenta previa, which can be confirmed through ultrasonography. Amniocentesis is not performed on a woman who is experiencing bleeding. In the event of an imminent delivery, the fetus is presumed to have immature lungs at this gestational age, and the mother is given corticosteroids to aid in fetal lung maturity. A CST is not performed at a preterm gestational age. Furthermore, bleeding is a contraindication to a CST. Internal fetal monitoring is also contraindicated in the presence of bleeding.

15. A woman arrives at the emergency department with complaints of bleeding and cramping. The initial nursing history is significant for a last menstrual period 6 weeks ago. On sterile speculum examination, the primary care provider finds that the cervix is closed. The anticipated plan of care for this woman would be based on a probable diagnosis of which type of spontaneous abortion? a. Incomplete b. Inevitable c. Threatened d. Septic

ANS: C A woman with a threatened abortion has spotting, mild cramps, and no cervical dilation. A woman with an incomplete abortion would have heavy bleeding, mild-to-severe cramping, and cervical dilation. An inevitable abortion demonstrates the same symptoms as an incomplete abortion: heavy bleeding, mild-to-severe cramping, and cervical dilation. A woman with a septic abortion has malodorous bleeding and typically a dilated cervix.

20. What condition indicates concealed hemorrhage when the client experiences abruptio placentae? a. Decrease in abdominal pain b. Bradycardia c. Hard, boardlike abdomen d. Decrease in fundal height

ANS: C Concealed hemorrhage occurs when the edges of the placenta do not separate. The formation of a hematoma behind the placenta and subsequent infiltration of the blood into the uterine muscle results in a very firm, boardlike abdomen. Abdominal pain may increase. The client will have shock symptoms that include tachycardia. As bleeding occurs, the fundal height increases.

6. A woman arrives for evaluation of signs and symptoms that include a missed period, adnexal fullness, tenderness, and dark red vaginal bleeding. On examination, the nurse notices an ecchymotic blueness around the woman's umbilicus. What does this finding indicate? a. Normal integumentary changes associated with pregnancy b. Turner sign associated with appendicitis c. Cullen sign associated with a ruptured ectopic pregnancy d. Chadwick sign associated with early pregnancy

ANS: C Cullen sign, the blue ecchymosis observed in the umbilical area, indicates hematoperitoneum associated with an undiagnosed ruptured intraabdominal ectopic pregnancy. Linea nigra on the abdomen is the normal integumentary change associated with pregnancy and exhibits a brown pigmented, vertical line on the lower abdomen. Turner sign is ecchymosis in the flank area, often associated with pancreatitis. A Chadwick sign is a blue-purple cervix that may be seen during or around the eighth week of pregnancy.

During a prenatal visit a nurse is explaining dietary management to a woman with pregestational diabetes. The nurse evaluates that teaching has been effective when the woman states: a. "I will need to eat 600 more calories per day because I am pregnant." b. "I can continue with the same diet as before pregnancy as long as it is well balanced." c. "Diet and insulin needs change during pregnancy." d. "I will plan my diet based on results of urine glucose testing.".

ANS: C Diet and insulin needs change during the pregnancy in direct correlation to hormonal changes and energy needs. In the third trimester, insulin needs may double or even quadruple. The diet is individualized to allow for increased fetal and metabolic requirements, with consideration of such factors as prepregnancy weight and dietary habits, overall health, ethnic background, lifestyle, stage of pregnancy, knowledge of nutrition, and insulin therapy. Energy needs are usually calculated on the basis of 30 to 35 calories per kilogram of ideal body weight. Dietary management during a diabetic pregnancy must be based on blood, not urine, glucose changes

10. The management of the pregnant client who has experienced a pregnancy loss depends on the type of miscarriage and the signs and symptoms. While planning care for a client who desires outpatient management after a first-trimester loss, what would the nurse expect the plan to include? a. Dilation and curettage (D&C) b. Dilation and evacuation (D&E) c. Misoprostol d. Ergot products

ANS: C Outpatient management of a first-trimester loss is safely accomplished by the intravaginal use of misoprostol for up to 2 days. If the bleeding is uncontrollable, vital signs are unstable, or signs of infection are present, then a surgical evacuation should be performed. D&C is a surgical procedure that requires dilation of the cervix and scraping of the uterine walls to remove the contents of pregnancy. This procedure is commonly performed to treat inevitable or incomplete abortion and should be performed in a hospital. D&E is usually performed after 16 weeks of pregnancy. The cervix is widely dilated, followed by removal of the contents of the uterus. Ergot products such as Methergine or Hemabate may be administered for excessive bleeding after miscarriage.

Metabolic changes throughout pregnancy that affect glucose and insulin in the mother and the fetus are complicated but important to understand. Nurses should know that: a. Insulin crosses the placenta to the fetus only in the first trimester, after which the fetus secretes its own b. Women with insulin-dependent diabetes are prone to hyperglycemia during the first trimester because they are consuming more sugar c. During the second and third trimesters, pregnancy exerts a diabetogenic effect that ensures an abundant supply of glucose for the fetus d. Maternal insulin requirements steadily decline during pregnancy

ANS: C Pregnant women develop increased insulin resistance during the second and third trimesters. Insulin never crosses the placenta; the fetus starts making its own around the tenth week. As a result of normal metabolic changes during pregnancy, insulin-dependent women are prone to hypoglycemia (low levels). Maternal insulin requirements may double or quadruple by the end of pregnancy.

8. A woman who is 30 weeks of gestation arrives at the hospital with bleeding. Which differential diagnosis would not be applicable for this client? a. Placenta previa b. Abruptio placentae c. Spontaneous abortion d. Cord insertion

ANS: C Spontaneous abortion is another name for miscarriage; it occurs, by definition, early in pregnancy. Placenta previa is a well-known reason for bleeding late in pregnancy. The premature separation of the placenta (abruptio placentae) is a bleeding disorder that can occur late in pregnancy. Cord insertion may cause a bleeding disorder that can also occur late in pregnancy.

6. A 30-year-old gravida 3, para 2-0-0-2 is at 18 weeks of gestation. What screening test should be suggested to her? a. Biophysical profile b. Chorionic villi sampling c. Maternal serum alpha-fetoprotein (MSAFP) screening d. Screening for diabetes mellitus

ANS: C The biochemical assessment MSAFP test is performed from week 15 to week 20 of gestation (weeks 16 to 18 are ideal). A biophysical profile is a method of biophysical assessment of fetal well-being in the third trimester. Chorionic villi sampling is a biochemical assessment of the fetus that should be performed from the tenth to twelfth weeks of gestation. Screening for diabetes mellitus begins with the first prenatal visit.

7. The nurse who elects to practice in the area of women's health must have a thorough understanding of miscarriage. Which statement regarding this condition is most accurate? a. A miscarriage is a natural pregnancy loss before labor begins. b. It occurs in fewer than 5% of all clinically recognized pregnancies. c. Careless maternal behavior, such as poor nutrition or excessive exercise, can be a factor in causing a miscarriage. d. If a miscarriage occurs before the 12th week of pregnancy, then it may be observed only as moderate discomfort and blood loss.

ANS: D Before the sixth week, the only evidence might be a heavy menstrual flow. After the 12th week, more severe pain, similar to that of labor, is likely. Miscarriage is a natural pregnancy loss, but it occurs, by definition, before 20 weeks of gestation, before the fetus is viable. Miscarriages occur in approximately 10% to 15% of all clinically recognized pregnancies. Miscarriages can be caused by a number of disorders or illnesses outside the mother's control or knowledge.

When evaluating the preterm infant, the nurse understands that compared with the term infant, what information is important for the nurse to understand? a.Few blood vessels visible through the skin b.More subcutaneous fat c.Well-developed flexor muscles d.Greater surface area in proportion to weight

ANS: D Preterm infants have greater surface area in proportion to their weight. More subcutaneous fat and well-developed muscles are indications of a more mature infant.

A client is diagnosed with gestational hypertension and is receiving magnesium sulfate. Which finding would the nurse interpret as indicating a therapeutic level of medication? A) Urinary output of 20 mL per hour B) Respiratory rate of 10 breaths/minute C) Deep tendons reflexes 2+ D) Difficulty in arousing

Ans: C With magnesium sulfate, deep tendon reflexes of 2+ would be considered normal and therefore a therapeutic level of the drug. Urinary output of less than 30 mL, a respiratory rate of less than 12 breaths/minute, and a diminished level of consciousness would indicate magnesium toxicity.

The nurse is reviewing the laboratory test results of a pregnant client. Which one of the following findings would alert the nurse to the development of HELLP syndrome? A) Hyperglycemia B) Elevated platelet count C) Leukocytosis D) Elevated liver enzymes

Ans: D HELLP is an acronym for hemolysis, elevated liver enzymes, and low platelets. Hyperglycemia or leukocytosis is not a part of this syndrome.

A nurse is assessing a pregnant woman with gestational hypertension. Which of the following would lead the nurse to suspect that the client has developed severe preeclampsia? A) Urine protein 300 mg/24 hours B) Blood pressure 150/96 mm Hg C) Mild facial edema D) Hyperreflexia

Ans: D Severe preeclampsia is characterized by blood pressure over 160/110 mm Hg, urine protein levels greater than 500 mg/24 hours and hyperreflexia. Mild facial edema is associated with mild preeclampsia.

With regard to skeletal injuries sustained by a neonate during labor or birth, nurses should be aware that: A. A newborn's skull is still forming and fractures fairly easily. B. Unless a blood vessel is involved, linear skull fractures heal without special treatment. C. Clavicle fractures often need to be set with an inserted pin for stability. D. Other than the skull, the most common skeletal injuries are to leg bones.

B A. Incorrect: Because the newborn skull is flexible, considerable force is required to fracture it. B. Correct: About 70% of neonatal skull fractures are linear. C. Incorrect: Clavicle fractures need no special treatment. D. Incorrect: The clavicle is the bone most often fractured during birth.

When planning care for an infant with a fractured clavicle, the nurse should recognize that in addition to gentle handling: A. Prone positioning will facilitate bone alignment. B. No special treatment is necessary. C. Parents should be taught range of motion exercises. D. The shoulder should be immobilized with a splint.

B A. Incorrect: Fractures in newborns generally heal rapidly. Except for gentle handling, no accepted treatment for a fractured clavicle exists. B. Correct: Fractures in newborns generally heal rapidly. Except for gentle handling, no accepted treatment for a fractured clavicle exists. C. Incorrect: Movement should be limited, and the infant should be gently handled. It is not necessary to perform range of motion exercises on the infant. D. Incorrect: A fractured clavicle does not require immobilization with a splint.

A client is admitted with vaginal bleeding at approximately 10 weeks of gestation. Her fundal height is 13 cm. Which potential problem should be investigated? a. Placenta previa b. Hydatidiform mole c. Abruptio placentae d. Disseminated intravascular coagulation (DIC)

B ~ Gestational trophoblastic disease (hydatidiform mole) is usually detected in the first trimester of pregnancy. The frequency of this condition is highest at both ends of a woman's reproductive life. Placenta previa usually occurs in the third trimester. Painless uterine bleeding is the classic symptom. Abruptio placentae usually occurs in the third trimester. Painful uterine bleeding is the classic symptom. DIC is a life-threatening complication of abruptio placentae, in which procoagulation and anticoagulation factors are simultaneously activated.

Which data found on a clients health history would place her at risk for an ectopic pregnancy? a. Ovarian cyst 2 years ago b. Recurrent pelvic infections c. Use of oral contraceptives for 5 years d. Heavy menstrual flow of 4 days duration

B ~ Infection and subsequent scarring of the fallopian tubes prevent normal movement of the fertilized ovum into the uterus for implantation. Ovarian cysts do not cause scarring of the fallopian tubes. Oral contraceptives do not increase the risk for ectopic pregnancies. Heavy menstrual flow of 4 days duration will not cause scarring of the fallopian tubes, which is the main risk factor for ectopic pregnancies.

Which assessment finding suggests that your laboring client's blood magnesium level is too high? a. Hyperactive reflexes b. Absent reflexes c. Generalized seizure d. Urine output of 60 mL/hr

B ~ Magnesium acts as a central nervous system depressant by blocking neuromuscular transmission. Assessment of the deep tendon reflexes is an indication of the level of CNS depression. Absent reflexes indicates magnesium toxicity; hyperactive reflexes, generalized seizure, and urine output of 60 mL/hr are not symptoms of magnesium toxicity.

A health care provider reports to the labor nurse that a patient is being transferred from the clinic directly to the hospital with possible preeclampsia. What is the nurse's priority action when the patient is admitted? a. Obtain the patients weight. b. Take the patients vital signs. c. Start an IV with lactated Ringers at 75 mL/hr. d. Ask support persons to leave the birthing room.

B ~ The hallmark signs of preeclampsia are hypertension and proteinuria. These parameters must be evaluated first. Obtaining the patients weight may indicate excess fluid gain, but fluid retention does not occur in all cases of preeclampsia. An IV will be beneficial; however, assessment precedes implementation in this case to obtain baseline data. Promoting a nonstimulating environment can help decrease blood pressure; however, loss of support during this frightening time can increase anxiety in this initial assessment phase and actually increase the patient's blood pressure.

A labor and birth nurse receives a call from the laboratory regarding a preeclamptic patient receiving an IV infusion of magnesium sulfate. The laboratory technician reports that the patients magnesium level is 7.6 mg/dL. What is the nurses priority action? a. Stop the infusion of magnesium. b. Assess the patients respiratory rate. c. Assess the patients deep tendon reflexes. d. Notify the health care provider of the magnesium level.

B ~ The therapeutic serum level for magnesium is 4 to 8 mg/dL although it is elevated in terms of normal lab values. Adverse reactions to magnesium sulfate usually occur if the serum level becomes too high. The most important is CNS depression, including depression of the respiratory center. Magnesium is excreted solely by the kidneys, and the reduced urine output that often occurs in preeclampsia allows magnesium to accumulate to toxic levels in the woman. Frequent assessment of serum magnesium levels, deep tendon reflexes, respiratory rate, and oxygen saturation can identify CNS depression before it progresses to respiratory depression or cardiac dysfunction. Monitoring urine output identifies oliguria that would allow magnesium to accumulate and reach excessive levels. Discontinue magnesium if the respiratory rate is below 12 breaths/min, a low pulse oximeter level (<95%) persists, or deep tendon reflexes are absent. Additional magnesium will make the condition worse.

The maternity nurse is preparing for the admission of a pt in the third trimester of pregnancy who is experiencing vaginal bleeding & has a suspected diagnosis of placenta previa. The nurse reviews the primary health care provider's prescriptions & should question which prescription? A. Prepare the client for an ultrasound. B. Obtain equipment for a manual pelvic examination. C. Prepare to draw a hemoglobin and hematocrit blood sample. D. Obtain equipment for external electronic fetal heart rate monitoring.

B: would never do a vag exam on a pt w vag bleeding of unknown cause

Providing care for the neonate born to a mother who abuses substances can present a challenge for the health care team. Nursing care for this infant requires a multisystem approach. The first step in the provision of this care is: A. Pharmacologic treatment B. Reduction of environmental stimuli C. Neonatal abstinence syndrome scoring D. Adequate nutrition and maintenance of fluid and electrolyte balance

C A. Incorrect: Pharmacologic treatment is based on the severity of withdrawal symptoms. Symptoms are determined by using a standard assessment tool. Medications of choice are morphine, phenobarbital, diazepam, or diluted tincture of opium. B. Incorrect: Swaddling, holding, and reducing environmental stimuli are essential in providing care to the infant who is experiencing withdrawal. These nursing interventions are appropriate for the infant who displays central nervous system disturbances. C. Correct: Neonatal abstinence syndrome (NAS) is the term used to describe the cohort of symptoms associated with drug withdrawal in the neonate. The Neonatal Abstinence Scoring System evaluates CNS, metabolic, vasomotor, respiratory, and gastrointestinal disturbances. This evaluation tool enables the care team to develop an appropriate plan of care. The infant is scored throughout the length of stay and the treatment plan is adjusted accordingly. D. Incorrect: Poor feeding is one of the GI symptoms common to this client population. Fluid and electrolyte balance must be maintained and adequate nutrition provided. These infants often have a poor suck reflex and may need to be fed via gavage.

Near the end of the first week of life, an infant who has not been treated for any infection develops a copper-colored, maculopapular rash on the palms and around the mouth and anus. The newborn is showing signs of: A. Gonorrhea B. Herpes simplex virus infection C. Congenital syphilis D. HIV

C A. Incorrect: The rash is indicative of congenital syphilis. The lesions may extend over the trunk and extremities. B. Incorrect: The rash is indicative of congenital syphilis. The lesions may extend over the trunk and extremities. C. Correct: The rash is indicative of congenital syphilis. The lesions may extend over the trunk and extremities. D. Incorrect: The rash is indicative of congenital syphilis. The lesions may extend over the trunk and extremities.

An infant was born 2 hours ago at 37 weeks of gestation, weighing 4.1 kg. The infant appears chubby with a flushed complexion and is very tremulous. The tremors are most likely the result of: A. Birth injury B. Hypocalcemia C. Hypoglycemia D. Seizures

C A. Incorrect: This infant is macrosomic and at risk for hypoglycemia. The description is indicative of a macrocosmic infant. The tremors are jitteriness that is associated with hypoglycemia. B. Incorrect: This infant is macrosomic and at risk for hypoglycemia. The description is indicative of a macrocosmic infant. The tremors are jitteriness that is associated with hypoglycemia. C. Correct: Hypoglycemia is common in the macrosomic infant. Signs of hypoglycemia include jitteriness, apnea, tachypnea, and cyanosis. D. Incorrect: This infant is macrosomic and at risk for hypoglycemia. The description is indicative of a macrocosmic infant. The tremors are jitteriness that is associated with hypoglycemia.

During assessment of the hospitalized client who is​ preeclamptic, the nurse notes a urine specific gravity of 1.050 after the most recent voiding. The nurse should monitor for which​ condition? A. Nonelong dash—this is a normal finding B. Urinary tract infection C. Oliguria and proteinuria D. Glycosuria

C ​Rationale: Renal damage or failure can occur in clients with preeclampsia. The specific gravity of the urine should be monitored. Laboratory findings of specific gravity over 1.040 correlate with oliguria and​ proteinuria, both of which are associated with renal damage. A urine specific gravity of 1.050 is not a normal finding​ (normal range 1.005 to​ 1.030). A specific gravity of 1.050 does not indicate glycosuria or a urinary tract infection.

A client with preeclampsia is being treated with bed rest and intravenous magnesium sulfate. The drug classification of this medication is a: a. diuretic. b. tocolytic. c. anticonvulsant. d. antihypertensive.

C ~ Anticonvulsant drugs act by blocking neuromuscular transmission and depress the central nervous system to control seizure activity. Diuresis is a therapeutic response to magnesium sulfate. A tocolytic drug slows the frequency and intensity of uterine contractions but is not used for that purpose in this scenario. Decreased peripheral blood pressure is a therapeutic response (side effect) of the anticonvulsant magnesium sulfate.

Which finding in the exam of a client with a diagnosis of threatened abortion would change the diagnosis to inevitable abortion? a. Presence of backache b. Rise in hCG level c. Clear fluid from vagina d. Pelvic pressure

C ~ Clear fluid from the vagina indicates rupture of the membranes. Abortion is usually inevitable (cannot be stopped) when the membranes rupture, the presence of backache and pelvic pressure are common symptoms in threatened abortion, and a rise in the hCG level is consistent with a viable pregnancy.

Which finding in the assessment of a client following an abruption placenta could indicate a major complication? a. Urine output of 30 mL in 1 hour b. Blood pressure of 110/60 mm Hg c. Bleeding at IV insertion site d. Respiratory rate of 16 breaths/min

C ~ DIC is a life-threatening defect in coagulation that may occur following abruptio placentae. DIC allows excess bleeding from any vulnerable area such as IV sites, incisions, gums, or nose. A urine output of 30 mL in 1 hour, blood pressure of 110/60 mm Hg, and respiratory rate of 16 breaths/min are normal findings in a postpartum client.

What is the priority nursing intervention for the client who has had an incomplete abortion? a. Methylergonovine (Methergine), 0.2 mg IM b. Preoperative teaching for surgery c. Insertion of IV line for fluid replacement d. Positioning of client in left side-lying position

C ~ Initial treatment of an incomplete abortion should be focused on stabilizing the client's cardiovascular state. Methylergonovine would be administered after surgical treatment, preoperative teaching is not a priority until the client is stabilized, and the left side-lying position provides no benefit to the client in this situation.

Which assessment in a client diagnosed with preeclampsia who is taking magnesium sulfate would indicate a therapeutic level of medication? a. Drowsiness b. Urinary output of 20 mL/hr c. Normal deep tendon reflexes d. Respiratory rate of 10 to 12 breaths/min

C ~ Magnesium sulfate is administered for preeclampsia to reduce the risk of seizures from cerebral irritability. Hyperreflexia (deep tendon reflexes above normal) is a symptom of cerebral irritability. If the dosage of magnesium sulfate is effective, reflexes should decrease to normal or slightly below normal levels. Drowsiness is another sign of CNS depression from magnesium toxicity. A urinary output of 20 mL/hr is not adequate output. A respiratory rate of 10 to 12 breaths/min is too slow and could be indicative of magnesium toxicity.

In addition to obtaining vital signs and FHT, what is a priority for the client with placenta previa? a. Determining cervical dilation b. Monitoring uterine contractions c. Estimating blood loss d. Starting a Pitocin drip

C ~ Nursing assessments for the client with placenta previa focus on determining the amount of blood loss. The nurse does not perform vaginal exams on a client with placenta previa because of the risk of perforating the placenta, the client may or may not be experiencing contractions, and induction is not indicated for a client with placenta previa.

What should the nurse recognize as evidence that the client is recovering from preeclampsia? a. 1+ protein in urine b. 2+ pitting edema in lower extremities c. Urine output >100 mL/hr d. Deep tendon reflexes +2

C ~ Rapid reduction of the edema associated with preeclampsia results in urinary output of 4 to 6 L/day as interstitial fluids shift back to the circulatory system. 1+ protein in urine and 2+ pitting edema in lower extremities are signs of continuing preeclampsia. Deep tendon reflexes are not a reliable sign, especially if the client has been treated with magnesium.

A 17-year-old primigravida has gained 4 pounds since her last prenatal visit. Her blood pressure is 140/92 mm Hg. The most important nursing action is to: a. advise her to cut down on fast foods that are high in fat. b. caution her to avoid salty foods and to return in 2 weeks. c. assess weight gain, location of edema, and urine for protein. d. recommend she stay home from school for a few days to reduce stress.

C ~ The nurse should further assess the client for hypertension, generalized edema, and proteinuria, which are classic signs of pregnancy-induced hypertension. Cutting down on fast foods will not relieve the symptoms of pregnancy-induced hypertension. She is at risk for pregnancy-induced hypertension and should be evaluated at this visit. Rest may be the treatment at first, but she needs further assessment to determine if pregnancy-induced hypertension is the problem.

The labor and birth nurse is reviewing the risk factors for placenta previa with a group of nursing students. The nurse determines that the students understood the discussion when they identify which patient being at the highest risk for developing a placenta previa? a. Female fetus, Mexican-American, primigravida b. Male fetus, Asian-American, previous preterm birth c. Male fetus, African-American, previous cesarean section d. Female fetus, European-American, previous spontaneous abortion

C ~ The rate of placenta previa is increasing. It is more common in older women, multiparous women, women who have had cesarean births, and women who had suction curettage for an induced or spontaneous abortion. It is also more likely to recur if a woman has had a placenta previa. African or Asian ethnicity also increases the risk. Cigarette smoking and cocaine use are personal habits that add to a woman's risk for a previa. Previa is more likely if the fetus is male. The Mexican-American primipara has no risk factors for developing a placenta previa. The Asian-American multipara has two risk factors for developing a previa. The African-American multipara has three risk factors for developing a previa. The European-American multigravida has one risk factor for developing a placenta previa.

Which finding on a prenatal visit at 10 weeks might suggest a hydatidiform mole? a. Blood pressure of 120/80 mm Hg b. Complaint of frequent mild nausea c. Fundal height measurement of 18 cm d. History of bright red spotting for 1 day weeks ago

C ~ The uterus in a hydatidiform molar pregnancy is often larger than would be expected on the basis of the duration of the pregnancy. A client with a molar pregnancy may have early-onset, pregnancy-induced hypertension. Nausea increases in a molar pregnancy because of the increased production of human chorionic gonadotropin (hCG). The history of bleeding is normally described as being of a brownish color.

A patient presents to labor and birth with complaints of persistent acute back pain at 36 weeks gestation. The nursing assessment reveals a taught abdomen, fundal height at 40 cm, and late decelerations, with an FHR range of 124 to 128 bpm. The nurse will implement the protocol for which obstetric condition? a. Placenta previa b. Hypovolemic shock c. Abruptio placentae or abruption d. DIC

C ~ There are five classic signs and symptoms of abruptio placentae and include the following: bleeding, which may be evident vaginally or be concealed behind the placenta; uterine tenderness, which may be localized at the site of the abruption; uterine irritability, with frequent low-intensity contractions and poor relaxation between contractions; abdominal or low back pain that may be described as aching or dull; and high uterine resting tone identified with the use of an intrauterine pressure catheter. Additional signs include nonreassuring FHR patterns, signs of hypovolemic shock, and fetal death. With a placenta previa there is bright red and painless bleeding. Hypovolemic shock can result from an abruption; however, if the protocol for shock is initiated, some of the blood work that can confirm an abruption will be omitted (e.g., a Kleihauer-Betke test). DIC can result from an abruption. First, look for the cause.

A macrosomic infant is born after a difficult, forceps-assisted delivery. After stabilization, the infant is weighed, and the birth weight is 4550 g (9 pounds, 6 ounces). The nurse's most appropriate action is to: A. Leave the infant in the room with the mother B. Take the infant immediately to the nursery C. Perform a gestational age assessment to determine whether the infant is large for gestational age D. Monitor blood glucose levels frequently and observe closely for signs of hypoglycemia

D A. Incorrect: Macrosomic infants are at high risk for hypoglycemia after birth and need to be observed closely. This can be achieved in the mother's room with nursing interventions, depending on the condition of the fetus. It may be more appropriate for observation to occur in the nursery. B. Incorrect: Macrosomic infants are at high risk for hypoglycemia after birth and need to be observed closely. Observation may occur in the nursery or in the mother's room, depending on the condition of the fetus. C. Incorrect: Regardless of gestational age, this infant is macrosomic. Macrosomia is defined as fetal weight over 4000 g. Hypoglycemia affects many macrosomic infants. Blood glucose levels should be observed closely. D. Correct: This infant is macrosomic (over 4000 g) and is at high risk for hypoglycemia. Blood glucose levels should be monitored frequently, and the infant should be observed closely for signs of hypoglycemia.

The priority nursing diagnosis for a newborn diagnosed with a diaphragmatic hernia would be: A. Risk for impaired parent-infant attachment B. Imbalanced nutrition: less than body requirements C. Risk for infection D. Impaired gas exchange

D A. Incorrect: Although this issue may be a factor in providing care to a newborn with a diaphragmatic hernia, the priority nursing diagnosis relates to the oxygenation issues arising from the lung hypoplasia that occurs with diaphragmatic hernia. B. Incorrect: Although this issue may be a factor in providing care to a newborn with a diaphragmatic hernia, the priority nursing diagnosis relates to the oxygenation issues arising from the lung hypoplasia that occurs with diaphragmatic hernia. C. Incorrect: Although this issue may be a factor in providing care to a newborn with a diaphragmatic hernia, the priority nursing diagnosis relates to the oxygenation issues arising from the lung hypoplasia that occurs with diaphragmatic hernia. D. Correct: Herniation of the abdominal viscera into the thoracic cavity may cause severe respiratory distress and represent a neonatal emergency. Oxygen therapy, mechanical ventilation, and the correction of acidosis are necessary in infants with large defects.

What bacterial infection is definitely decreasing because of effective drug treatment? A. Escherichia coli infection B. Tuberculosis C. Candidiasis D. Group B streptococcal infection

D A. Incorrect: E. coli may be increasing, perhaps because of the increasing use of ampicillin (resulting in a more virulent E. coli resistant to the drug). Group B streptococcus has been beaten back by penicillin. B. Incorrect: Tuberculosis is increasing in the United States and in Canada. Group B streptococcus has been beaten back by penicillin. C. Incorrect: Candidiasis is a fairly benign fungal infection. Group B streptococcus has been beaten back by penicillin. D. Correct: Penicillin has significantly decreased the incidence of group B streptococcal infection.

What finding supports the diagnosis of pathologic jaundice? A. Serum bilirubin concentrations greater than 2 mg/dl in cord blood B. Serum bilirubin levels increasing more than 1 mg/dl in 24 hours C. Serum bilirubin levels greater than 10 mg/dl in a full-term newborn D. Clinical jaundice evident within 24 hours of birth

D A. Incorrect: Serum bilirubin concentrations greater than 4 mg/dl in cord blood would support a diagnosis of pathologic jaundice. B. Incorrect: Total serum bilirubin levels that increase by more than 5 mg/dl in 24 hours would support a diagnosis of pathologic jaundice. C. Incorrect: A serum bilirubin level in a preterm newborn that exceeds 10 mg/dl would support a diagnosis of pathologic jaundice. D. Correct: Clinical jaundice evident within 24 hours of birth would support a diagnosis of pathologic jaundice.

As a home care nurse, you are visiting a 5-day-old male infant for a scheduled follow-up appointment to ensure that he is responding to home phototherapy for treatment of jaundice. Based on the diagnosis of hyperbilirubinemia, you are aware that the development of acute bilirubin encephalopathy is a risk for this infant. This disease process occurs after the bilirubin level has peaked. After completing a thorough assessment and obtaining a history from the parents, you recognize that this infant is in the first phase of encephalopathy when he exhibits: A. A high-pitched cry B. Severe muscle spasms (opisthotonos) C. Fever and seizures D. Hypotonia, lethargy, and poor suck

D A. Incorrect: Should the infant display symptoms such as a high-pitched cry, severe muscle spasms, hyperreflexia, or arching of the back, the nurse should be aware that the baby has progressed beyond the more subtle signs of the first phase. Medical attention is necessary immediately. B. Incorrect: Should the infant display symptoms such as a high-pitched cry, severe muscle spasms, hyperreflexia, or arching of the back, the nurse should be aware that the baby has progressed beyond the more subtle signs of the first phase. Medical attention is necessary immediately. C. Incorrect: Symptoms may progress from the subtle indications of the first phase to fever and seizures in as little as 24 hours. Only about half of these infants survive and will have permanent sequelae including auditory deficiencies, intellectual deficits, and movement abnormalities. D. Correct: The early and most subtle symptoms of bilirubin encephalopathy include hypotonia, lethargy, poor suck, and depressed or absent Moro reflex.

A pregnant woman at 37 weeks of gestation has had ruptured membranes for 26 hours. A cesarean section is performed for failure to progress. The fetal heart rate before birth is 180 beats/min with limited variability. At birth, the newborn has Apgar scores of 6 and 7 at 1 and 5 minutes and is noted to be pale and tachypneic. Based on the maternal history, the cause of this newborn's distress is most likely to be: A. Hypoglycemia B. Phrenic nerve injury C. Respiratory distress syndrome D. Sepsis

D A. Incorrect: The prolonged rupture of membranes is the most indicative clinical cue to this infant's condition. An FHR of 180 beats/min is also indicative. This infant is at high risk for sepsis. B. Incorrect: The prolonged rupture of membranes is the most indicative clinical cue to this infant's condition. An FHR of 180 beats/min is also indicative. This infant is at high risk for sepsis. C. Incorrect: The prolonged rupture of membranes is the most indicative clinical cue to this infant's condition. An FHR of 180 beats/min is also indicative. This infant is at high risk for sepsis. D. Correct: The prolonged rupture of membranes and the tachypnea (before and after birth) both suggest sepsis.

In order to provide comprehensive newborn care, the nurse should understand that kernicterus occurs if: A. The kidney excretes bilirubin. B. Bilirubin collects in the liver. C. Bilirubin deposits are concentrated in the cardiac muscle. D. Bilirubin deposits are in the brain.

D A. Incorrect: The term kernicterus is synonymous with bilirubin encephalopathy. It is caused by the deposition of bilirubin in the brain. B. Incorrect: The term kernicterus is synonymous with bilirubin encephalopathy. It is caused by the deposition of bilirubin in the brain. C. Incorrect: The term kernicterus is synonymous with bilirubin encephalopathy. It is caused by the deposition of bilirubin in the brain. D. Correct: Kernicterus describes the chronic and permanent results of bilirubin toxicity.

HIV may be perinatally transmitted: A. Only in the third trimester from the maternal circulation B. By a needlestick injury at birth from unsterile instruments C. Only through the ingestion of amniotic fluid D. Through the ingestion of breast milk from an infected mother

D A. Incorrect: Transmission of HIV from the mother to the infant may occur transplacentally at various gestational ages. Transmission close to or at the time of birth is thought to account for 50% to 80% of cases.B. Incorrect: Transmission of HIV may occur through the placenta from the mother to the fetus or through breast milk postnatally.C. Incorrect: Transmission of HIV may occur through the placenta from the mother to the fetus or through breast milk postnatally.D. Correct: Postnatal transmission of HIV through breastfeeding may occur.

A pt arrives at the health care clinic & tells the nurse that her last menstrual period was 9 weeks ago. The pt tells the nurse that a home pregnancy test was positive but that she began to have mild cramps & is now having moderate vaginal bleeding. On physical examination of the pt, it is noted that she has a dilated cervix. Which statement, if made by the pt, indicates that the pt is interpreting the situation correctly? A. "I will need to remain on bed rest for 2 weeks." B. "I will need to take a full course of antibiotic treatment." C. "I will need to take tocolytic medication to halt the labor process." D. "I will need to prepare myself and my family for the loss of this pregnancy."

D THIS IS AN INEVITABLE MISCARRIAGE

The nurse is providing care to a client diagnosed with preeclampsia during the antepartum period. Which information should the nurse provide to the​ client? A. "Your activity and diet will be​ restricted." B. "You will be prescribed an antihypertensive​ drug." C. "You should lie on your right side when you are​ resting." D. "You may need to be​ hospitalized."

D ​Rationale: Antepartum management for preeclampsia may include hospitalization to evaluate​ new-onset maternal and fetal conditions. There is no evidence in favor of antihypertensive medication unless maternal BP is in the severe range. Activity restriction and dietary modifications do not alter the course or outcome of preeclampsia. When at​ rest, the client may be encouraged to lie on her left side in order to maximize uterine and renal perfusion.

A pregnant client at 28 weeks of gestation is diagnosed with preeclampsia after reporting persistent headache and blurry vision. The​ client's blood pressure is​ 162/112 mmHg and hospitalization is recommended. The client asks the nurse if the baby will have to be delivered early. Which response best addresses the​ client's question? A. "I know that you are​ frightened, but right now what is most important is getting you to the hospital. You can discuss these concerns with your healthcare provider​ later." B. "Given that you have such severe symptoms at only 28 weeks of​ gestation, it is highly likely that your baby will be delivered​ prematurely." C. "Placing you in the hospital will help to ensure that we can increase the length of time before you​ deliver, allowing the baby to​ mature." D. "I am not sure right now. The healthcare provider will speak with you about it after considering the severity of the disease for you against the risks for your​ baby."

D ​Rationale: At this point in​ time, the​ nurse's best response is to honestly inform the client that he is not sure but that the healthcare provider will speak with the client about it after considering the severity of the disease for her against the risks for the baby. The nurse cannot ensure that hospitalization will increase the length of time before delivery nor can the nurse know that the baby will necessarily be delivered​ early, so this would not be the best response for the nurse to make. Telling the client that the primary concern is getting her to the hospital dismisses the current client concerns as not important and would not be the best response to the question.

The nurse is monitoring a client who is hospitalized at 30 weeks of gestation for preeclampsia. Which manifestation should the nurse recognize as indicating progression to​ eclampsia? A. Elevated liver enzymes B. Pulmonary edema C. Visual disturbances D. Seizures

D ​Rationale: Eclampsia is preeclampsia with the presence of seizures. Seizures would indicate progression to eclampsia. Visual​ disturbances, pulmonary​ edema, and elevated liver enzymes are all clinical manifestations of​ preeclampsia, but do not indicate progression to eclampsia.

A pregnant client at 31 weeks of gestation has been hospitalized due to worsening symptoms of preeclampsia. Which medication should the nurse anticipate administering to the client to reduce morbidity for the potentially preterm​ infant? A. Diuretic B. Antihypertensive C. Antibiotic D. Corticosteroid

D ​Rationale: Prior to 34​ weeks, corticosteroids may be administered to the mother to accelerate fetal lung development and reduce preterm infant morbidity. Diuretics and antibiotics are sometimes used with the premature infant but are not indicated for administration to the mother. Unless maternal blood pressure is in the severe​ range, there is no evidence in favor of the use of antihypertensive medication.

The doctor suspects that the client has an ectopic pregnancy. Which symptom is consistent with a diagnosis of ectopic pregnancy? a. Painless vaginal bleeding b. Abdominal cramping c. Throbbing pain in the upper quadrant d. Sudden, stabbing pain in the lower quadrant

D . The signs of an ectopic pregnancy are vague until the fallopian tube ruptures. The client will complain of sudden, stabbing pain in the lower quadrant that radiates down the leg or up into the chest. Painless vaginal bleeding is a sign of placenta previa, abdominal cramping is a sign of labor, and throbbing pain in the upper quadrant is not a sign of an ectopic pregnancy, making answers A, B, and C incorrect.

A placenta previa when the placental edge just reaches the internal os is called: a. total. b. partial. c. low-lying. d. marginal.

D ~ A placenta previa that does not cover any part of the cervix is termed marginal. With a total placenta previa, the placenta completely covers the os. With a partial previa, the lower border of the placenta is within 3 cm of the internal cervical os but does not completely cover the os. A complete previa is termed total. The placenta completely covers the internal cervical os.

A client with no prenatal care delivers a healthy male infant via the vaginal route, with minimal blood loss. During the labor period, vital signs were normal. At birth, significant maternal hypertension is noted. When the client is questioned, she relates that there is history of heart disease in her family but that she has never been treated for hypertension. Blood pressure is treated in the hospital setting and the client is discharged. The client returns at her scheduled 6-week checkup and is found to be hypertensive. Which type of hypertension do you think the client is exhibiting? a. Pregnancy-induced hypertension (PIH) b. Gestational hypertension c. Preeclampsia superimposed on chronic hypertension d. Undiagnosed chronic hypertension

D ~ Even though the client has no documented prenatal care or medical history, she does relate a family history that is positive for heart disease. Additionally, the clients blood pressure increased following birth and was treated in the hospital and resolved. Now the client appears at the 6-week checkup with hypertension. Typically, gestational hypertension resolves by the end of the 6-week postpartum period. The fact that this has not resolved is suspicious for undiagnosed chronic hypertension. There is no evidence to suggest that the client was preeclamptic prior to the birth.

Which assessment finding indicates the development of preeclampsia in the antepartum client? a. Slight edema of feet and ankles. b. Increased urine output c. Blood pressure of 128/80 mm Hg d. Weight gain of 3 pounds in 1 week

D ~ Generalized edema often occurs with preeclampsia. Edema may first manifest as a rapid weight gain. Normal weight gain in the second and third trimesters is 1 pound per week; slight edema of feet and ankles, increased urine output, and blood pressure of 128/80 mm Hg are normal findings in pregnancy.

The nurse is assessing a pregnant pt w type 1 diabetes mellitus about her understanding regarding changing insulin needs during pregnancy. The nurse determines that further teaching is needed if the pt makes which statement? A. "I will need to increase my insulin dosage during the first 3 months of pregnancy." B. "My insulin dose will likely need to be increased during the second and third trimesters." C. "Episodes of hypoglycemia are more likely to occur during the first 3 months of pregnancy." D. "My insulin needs should return to prepregnant levels within 7 to 10 days after birth if I am bottle-feeding."

a ("type 1 DM will be on insulin") Insulin needs are reduced in the first trimester due to increased insulin production by the pancreas and increased peripheral sensitivity to insulin. Insulin resistance begins as early as 14 to 16 weeks of gestation and continues to rise until it stabilizes during the last few weeks of pregnancy. During the first trimester maternal blood glucose levels are reduced and the insulin response to glucose is enhanced; therefore, this is when an episode of hypoglycemia is most likely to occur. For the nonbreastfeeding mother insulin levels return to normal within 7 to 10 days. Lactation uses maternal glucose; therefore, the mother's insulin requirements remain low during lactation. On completion of weaning the mother's prepregnancy insulin requirement is reestablished.

The nurse realizes that a neonate born at 34 weeks' gestation might not have enough surfactant, so the nurse should observe closely for which of the following?Select all that apply. a. Tachypnea b. Abdominal distention c. Jaundice d. Sternal retractions e. Jitteriness

a, d Explanation: Abdominal distention is not directly related to RDS. Jaundice is not directly related to RDS. Jitteriness is not directly related to RDS. Preterm infants lack adequate surfactant to keep their alveoli open during expiration. This can lead to development of respiratory distress syndrome (RDS), which would be evidenced by signs of respiratory distress, including sternal retractions. Preterm infants lack adequate surfactant to keep their alveoli open during expiration. This can lead to development of respiratory distress syndrome (RDS), which would be evidenced by signs of respiratory distress, including tachypnea.

The parents of a 28-weeks'-gestation neonate ask the nurse, "Why does he have to be fed through a tube in his mouth?" What is the best response by the nurse? a. "The baby's sucking, swallowing, and breathing are not coordinated yet." b. "The baby's stomach cannot digest formula at this time." c. "It helps to prevent thrush, an infection that could affect the baby's mouth." d. "It allows us to accurately determine the baby's intake."

a. "The baby's sucking, swallowing, and breathing are not coordinated yet." Explanation: Intake can be accurately assessed with gavage feedings but the concern is risk for aspiration. Neonates generally aren't able to effectively coordinate sucking, swallowing, and breathing until 34-36 weeks' gestation. If fed orally before that time, they are at greater risk of aspiration. Typically, they will be fed through a gavage tube until they are able to drink from a bottle or breastfeed. The stomach of a preterm infant can digest small amounts of formula or breast milk. Thrush is an oral yeast infection commonly caused during passage through the birth canal, and gavage feedings will not prevent it from occurring.

The nurse developing a plan of care for a postterm small-for-gestational-age (SGA) newborn would identify which assessment as the priority to monitor? a. Blood glucose levels b. Hemoglobin and hematocrit c. Total bilirubin levels d. Urinary output

a. Blood glucose levels The most common metabolic complication in the SGA newborn is hypoglycemia, which can produce central nervous system abnormalities and mental retardation if not corrected immediately. Urinary output, although important, is not the highest priority action; however, the postterm SGA newborn is typically dehydrated from placental dysfunction. Hemoglobin and hematocrit levels are monitored because the postterm SGA newborn exhibits polycythemia, although this also does not require immediate attention. The polycythemia contributes to increased bilirubin levels, usually beginning on the second day after delivery.

A nurse is caring for a 12-hour-old newborn. The nurse notes a yellow tint to the baby's skin and sclera. What laboratory test should the nurse anticipate being ordered? a. Direct Coombs' b. Blood culture c. Arterial blood gas (ABG) d. Blood glucose

a. Direct Coombs' Explanation:Blood glucose is not related to hyperbilirubinemia. Jaundice in an infant less than 24 hours old is often caused by Rh or ABO incompatibility. A direct Coombs' test determines the presence of maternal antibodies in the baby's blood. A blood culture is not related to hyperbilirubinemia. ABG's are not related to hyperbilirubinemia.

The nurse in the newborn nursery is planning for the admission of a large for gestational age (LGA) infant whose mother has gestational diabetes. In preparing to care for this infant, the nurse would obtain equipment to perform which diagnostic test? a. Heel stick blood glucose b. Rh and ABO blood typing c. Serum insulin levels d. Indirect and direct bilirubin levels

a. Heel stick blood glucose After birth, the most common problem in the LGA infant is hypoglycemia, especially if the mother is diabetic. At delivery when the umbilical cord is clamped and cut, maternal blood glucose supply is lost. The newborn continues to produce large amounts of insulin, which depletes the infant's blood glucose within the first hours after birth. If immediate identification and treatment of hypoglycemia are not performed, the newborn may suffer central nervous system damage caused by inadequate circulation of glucose to the brain. Serum insulin levels are not helpful because there is no intervention to decrease these levels to prevent hypoglycemia. There is no rationale for prescribing an Rh and ABO blood type unless the maternal blood type is O or Rh negative. Indirect and direct bilirubin levels are usually prescribed after the first 24 hours because jaundice is usually seen at 48 to 72 hours after birth.

An 8-pound 15-ounce baby born at 36 weeks' gestation would be described using which terminology? Select all that apply. a. Large for gestational age (LGA) b. Normal for gestational age c. Preterm d. Term e. Immature f. Postterm

a. Large for gestational age (LGA) c. Preterm A neonate born before the end of 37 weeks' gestation is considered preterm, regardless of weight. The large-for-gestational-age neonate is designated by a weight that is above the 90th percentile. A newborn weighing more than 8 pounds 14 ounces at any time is a large-for-gestational-age baby. A term newborn is older than 38 weeks, born between the beginning of week 38 and the end of week 41. A postterm newborn is born at week 42 or after. An immature neonate is born between 37 and 38 weeks' gestation. An average-for-gestational-age weight falls between the 11th and 89th percentiles.

Which nursing intervention is appropriate in the care of an infant with respiratory distress syndrome (RDS)? a. Maintain a neutral thermal environment. b. Perform chest physiotherapy twice a day. c. Perform a complete gestational age assessment. d. Suction meconium from airway as needed.

a. Maintain a neutral thermal environment. Explanation: Infants use additional oxygen and glucose when faced with cold stress. Infants with RDS are already compromised, so it is important to keep environmental temperatures stable to minimize their oxygen and glucose requirements. A complete assessment could increase oxygenation requirements even further. Chest physiotherapy might or might not be needed. There is no specific evidence in the question that meconium is present.

The parents of a preterm neonate ask why their baby gets cold so easily. The nurse responds with which explanation about preterm neonates? a. Preterm neonates have minimal body fat to retain body heat. b. Preterm neonates lose heat faster because they lie in a fetal position. c. Preterm neonates are able to shiver to produce body heat. d. Preterm neonates have blood vessels that are deep under the skin surface.

a. Preterm neonates have minimal body fat to retain body heat. Explanation: In general, infants are not able to shiver to produce body heat when they are cold. Preterm infants have minimal adipose tissue, so they lose heat more quickly through their skin. The skin of a neonate is thin, with blood vessels near the surface, which increases heat loss through the skin. Because they are weak and neurologically immature, they aren't able to lie in a tight fetal position, allowing greater exposure of the body to the air, which results in heat loss.

How would the nurse best explain the probable cause of jaundice to the parents of a 3-day-old newborn? a. The body is slow to get rid of the fetal red blood cells that have been destroyed b. An allergic response to the feedings c. The seepage of maternal Rh-negative blood into the neonate's bloodstream d. A temporary bile duct obstruction commonly found in newborns

a. The body is slow to get rid of the fetal red blood cells that have been destroyed After birth, fetal erythrocytes hemolyze, releasing bilirubin into the circulation; the immature liver cannot metabolize the bilirubin as rapidly as it is produced, resulting in physiological jaundice. Jaundice is not an allergic response; it is a physiological destruction of fetal red blood cells. Bile duct obstruction, which is not common in newborns, is not the cause of the jaundice. The newborn and mother have independent circulations, and Rh-negative blood does not enter the fetus's bloodstream. A problem may occur if the mother is sensitized, because her antibodies can enter the fetal circulation.

2. A 39-year-old primigravida thinks that she is about 8 weeks pregnant, although she has had irregular menstrual periods all her life. She has a history of smoking approximately one pack of cigarettes a day, but she tells you that she is trying to cut down. Her laboratory data are within normal limits. What diagnostic technique could be used with this pregnant woman at this time? a. Ultrasound examination b. Maternal serum alpha-fetoprotein (MSAFP) screening c. Amniocentesis d. Nonstress test (NST)

a. Ultrasound examination An ultrasound examination could be done to confirm the pregnancy and determine the gestational age of the fetus. It is too early in the pregnancy to perform MSAFP screening, amniocentesis, or NST. MSAFP screening is performed at 16 to 18 weeks of gestation, followed by amniocentesis if MSAFP levels are abnormal or if fetal/maternal anomalies are detected. NST is performed to assess fetal well-being in the third trimester.

A newborn's temperature is 97.4°F. What is the priority nursing intervention? a. Wrap the newborn in two warm blankets, and place a cap on the head. b. Take the newborn to the nursery, and observe for two hours. c. Reassess the temperature in four hours. d. Notify the physician or nurse practitioner immediately.

a. Wrap the newborn in two warm blankets, and place a cap on the head. Explanation: The physician can be notified if needed once the infant has been attended to. Observing the infant is an assessment and not a protective intervention. Reassessing the temperature in four hours is not a protective intervention. This newborn has a low temperature, and the nurse must intervene quickly to prevent complications related to hypothermia. Wrapping the baby in warm blankets and covering the head will help prevent heat loss through conduction, convection, and radiation, and are the most important initial interventions. A baby can lose a large amount of heat from his head, so keeping it covered will help stabilize the temperature.

If the fetus has already died but there is no bleeding or cramping then this is what type of miscarriage? a. missed miscarriage b. complete miscarriage c. threatened miscarriage d. inevitable miscarriage

a. missed miscarriage

The nurse assesses a newborn and obtains the following information: Left arm limp and extended; left hand internally rotated; positive grasp reflex bilaterally; no response on left side to Moro reflex. What is the most appropriate nursing intervention for this infant? Select all that apply. a. Prepare supplies for a cast application. b. Avoid positioning infant on left side. c. Immobilize the arm by securing the infant's sleeve to the shirt d. Provide passive range of motion exercises after 24 hours e. Assess for congenital hip dysplasia.

b, c Explanation: Congenital hip dysplasia is characterized by a clicking sound with hip rotation, while this infant has Erb-Duchenne's paralysis (Erb's palsy) of the left arm. The infant should not be positioned on the affected side. Passive range of motion is delayed until the 10th day to prevent further damage. Occasionally a splint may be applied, but a cast is not indicated. The arm may be secured by securing the infant's sleeve to the shirt or using a brace or splint.

A mother asks the neonatal nurse why her infant must be monitored so closely for hypoglycemia when her type 1 diabetes was in excellent control during the entire pregnancy. How would the nurse best respond? a. "A healthy newborn's glucose level drops after birth, so we're being especially cautious with your baby because of your diabetes." b. "Babies of mothers with diabetes have a higher-than-average insulin level because of the excess glucose received from the mother during pregnancy, so the glucose level may drop." c. "Babies of mothers with diabetes do not have large stores of glucose at birth, so it's difficult for them to maintain the blood glucose level within an acceptable range." d. "A newborn's pancreas produces an increased amount of insulin during the first day of birth, so we're checking to see whether hypoglycemia has occurred."

b. "Babies of mothers with diabetes have a higher-than-average insulin level because of the excess glucose received from the mother during pregnancy, so the glucose level may drop." The infant of a diabetic mother (IDM) produces a higher level of insulin in response to the increased maternal glucose level; after birth it takes several hours for the newborn to adjust to the loss of the maternal glucose. A healthy newborn's glucose level does not drop significantly after birth. A newborn's pancreas usually produces more insulin as a response to the maternal glucose level, but this response is not specific to the IDM. IDMs have the same glucose stores as other newborns; their responses to the loss of maternal glucose levels differ.

4. A 41-week pregnant multigravida presents in the labor and delivery unit after a nonstress test indicated that her fetus could be experiencing some difficulties in utero. Which diagnostic tool would yield more detailed information about the fetus? a. Ultrasound for fetal anomalies b. Biophysical profile (BPP) c. Maternal serum alpha-fetoprotein (MSAFP) screening d. Percutaneous umbilical blood sampling (PUBS)

b. Biophysical profile (BPP) Real-time ultrasound permits detailed assessment of the physical and physiologic characteristics of the developing fetus and cataloging of normal and abnormal biophysical responses to stimuli. BPP is a noninvasive, dynamic assessment of a fetus that is based on acute and chronic markers of fetal disease. An ultrasound for fetal anomalies would most likely have been performed earlier in the pregnancy. It is too late in the pregnancy to perform MSAFP screening. Also, MSAFP screening does not provide information related to fetal well-being. Indications for PUBS include prenatal diagnosis or inherited blood disorders, karyotyping of malformed fetuses, detection of fetal infection, determination of the acid-base status of a fetus with IUGR, and assessment and treatment of isoimmunization and thrombocytopenia in the fetus.

A woman diagnosed with gestational hypertension is at a prenatal appointment. During the assessment, the nurse is concerned that she is developing signs/symptoms that indicate that her gestational hypertension is worsening. What would the nurse have assessed that indicates that the condition is worsening? Select all that apply. a. Negative protein on dipstick of urine b. Blood pressure (BP) 165/120 mm Hg c. Denial of visual problems d. Complaints of headache for the last 12 hours

b. Blood pressure (BP) 165/120 mm Hg d. Complaints of headache for the last 12 hours Severe gestational hypertension or preeclampsia may be forms of progression of mild gestational hypertension. The blood pressure (BP) increases above 140/90 as does the proteinuria. The woman begins to have complaints of neurological symptoms. Elevated blood pressure and headaches are indicative of a worsening condition.

A nurse is assessing a neonate born 12 hours ago, and notes a yellow tint to the sclera. The nurse should read the medical record for what other assessment that is important to note at this time? a. Length of time prior to delivery that membranes ruptured b. Blood type and Rh factor of mother and newborn c. Most recent blood pressure d. Blood glucose

b. Blood type and Rh factor of mother and newborn Explanation: Blood glucose is not of concern with jaundice. This newborn has signs of jaundice, which include a yellow tint to the sclera and skin. Jaundice is considered pathologic if it occurs in the first 24 hours of life, when it is most often caused by Rh or ABO incompatibility. It would be important to assess both the mother's and newborn's blood type and Rh factor to determine if this could be causing the jaundice. A bilirubin level should also be obtained. Most recent blood pressure is not of concern with jaundice. Timing of membrane rupture is not of concern with jaundice.

Although the newborn was just cleaned and examined, the mother notes a red rash consisting of small papules on the face, chest, and back of the newborn. Which condition would the nurse recognize? a. Nevus flammeus b. Erythema toxicum c. Vernix caseosa d. Harlequin sign

b. Erythema toxicum Erythema toxicum is a benign, generalized, transient rash that is a reaction to the new environment in which a neonate finds itself. It disappears a short time after birth. It is not the harlequin sign, which is dilation of blood vessels on one side of the body resulting in red skin on one side and white skin on the other. It is not vernix caseosa, which is a thick, white, greasy substance that protects the skin in utero. It is not nevus flammeus, or port wine stain, which is a reddish-purple capillary angioma below the dermis.

The nurse must continually assess a preterm infant's temperature and provide appropriate nursing care because, unlike the full-term infant, the preterm infant has which limitation? a. Has a limited amount of pituitary hormones with which to control internal heat b. Has a limited supply of brown fat available to provide heat c. Cannot break down glycogen to glucose d. Cannot use shivering to produce heat

b. Has a limited supply of brown fat available to provide heat Because neonates are unable to shiver, they use the breakdown of brown fat to supply body heat; the preterm infant has a limited supply of brown fat available for this purpose. An inability to use shivering to produce heat is not specific to preterm neonates; all newborns are unable to use shivering to supply body heat. The breakdown of glycogen into glucose does not supply body heat. Pituitary hormones do not regulate body heat.

Which of the following would be a priority nursing intervention for a newborn experiencing hypothermia? a. Administering oxygen b. Monitoring for hypoglycemia c. Rapidly rewarming the newborn d. Starting phototherapy

b. Monitoring for hypoglycemia Explanation: The newborn reacts to hypothermia by burning brown fat to produce body heat. This process requires oxygen and glucose. When an infant experiences hypothermia, glucose and oxygen needs increase, and hypoglycemia can result. Newborns should be rewarmed slowly to prevent hypotension. The newborn might require oxygen administration, but the need should always be assessed first. Phototherapy is not indicated.

Which data would be most important for the nurse to note as part of an initial assessment of a newborn's history? a. Mother's age is 14. b. Mother received morphine sulfate 4 mg IV 20 minutes before delivery. c. Mother reports drinking a glass of wine with dinner each night. d. Mother's blood type is O negative.

b. Mother received morphine sulfate 4 mg IV 20 minutes before delivery. Explanation: Opioid analgesics cross the placenta and, if given close to delivery, can cause respiratory depression in the newborn, making this the priority item. Maternal drinking might warrant further investigation, but the priority at delivery is to establish and maintain an airway. Young maternal age has possible implications that might warrant further investigation, but the priority at delivery is to establish and maintain an airway. Maternal blood type O negative warrants follow-up, but the priority at delivery is to establish and maintain an airway.

A newborn male is admitted to the nursery 15 minutes after delivery. His skin is mottled, and mucous membranes are blue; he is active, and is wrapped in a blanket. The nurse should make which assessment as a priority? a. Umbilical cord for bleeding b. Patent airway c. Visible deformities d. Infant's temperature

b. Patent airway Explanation: Once airway and breathing are assessed, the nurse may check the umbilical cord for bleeding, measure temperature, and, finally, check for visible deformities. Once airway and breathing are assessed, the nurse may check the umbilical cord for bleeding, measure temperature, and, finally, check for visible deformities. Once airway and breathing are assessed, the nurse may check the umbilical cord for bleeding, measure temperature, and, finally, check for visible deformities. The highest priority after delivery is to maintain and support respiratory function. This infant is demonstrating initial signs of respiratory deficiency.

A nurse is admitting a baby to the nursery 30 minutes after delivery. Which information from the mother's history should be of greatest concern? a. Membranes ruptured 10 hours prior to delivery. b. Preexisting insulin-dependent diabetes mellitus. c. Received meperidine (Demerol) IV three hours prior to delivery. d. Marginal placenta previa.

b. Preexisting insulin-dependent diabetes mellitus. Explanation: A maternal history of diabetes increases the risk of hypoglycemia in the newborn, and this infant should be monitored closely. If the woman received meperidine (Demerol), most of the drug would be metabolized within three hours, and should not cause respiratory depression in the infant at delivery. A marginal placenta previa increases the mother's risk of bleeding during pregnancy, but should not cause significant complications in the newborn after delivery. Membranes ruptured greater than 24 hours prior to delivery increase the risk of infection for mother and infant.

In her 36th week of gestation, a client with type 1 diabetes delivers a 9 lb 10 oz (4366 g) infant via cesarean birth. Which condition is this infant at a high risk for developing? a. Increased intracranial pressure b. Respiratory distress syndrome c. Meconium ileus d. Physiological jaundice

b. Respiratory distress syndrome A large-for-gestational-age infant born at 36 weeks' gestation to a mother with diabetes may have immature lung tissue, which predisposes the newborn to respiratory distress. Meconium ileus is suggestive of cystic fibrosis, which is unrelated to maternal diabetes. Physiological jaundice manifests about 24 hours after birth, when fetal red blood cells begin to be subjected to hemolysis; this is unrelated to maternal diabetes. Increased intracranial pressure may be associated with birth injury or hydrocephalus; it is unrelated to maternal diabetes.

The nurse is assigned to a baby receiving phototherapy. Which assessment warrants further investigation by the nurse? a. Loose green stools b. Temperature 97.2°F c. Fine, red rash on trunk d. Yellow tint to the skin

b. Temperature 97.2°F Explanation: Loose green stools are expected findings with hyperbilirubinemia. A yellow tint to the skin is an expected finding with hyperbilirubinemia. Any temperature below 97.6°F is considered hypothermia, and requires immediate attention. A fine, raised red rash might appear on the infant's skin as a side effect of the phototherapy, and does not require intervention.

7. Maternal serum alpha-fetoprotein (MSAFP) screening indicates an elevated level. MSAFP screening is repeated and again is reported as higher than normal. What would be the next step in the assessment sequence to determine the well-being of the fetus? a. Percutaneous umbilical blood sampling (PUBS) b. Ultrasound for fetal anomalies c. Biophysical profile (BPP) for fetal well-being d. Amniocentesis for genetic anomalies

b. Ultrasound for fetal anomalies If MSAFP findings are abnormal, follow-up procedures include genetic counseling for families with a history of neural tube defect, repeated MSAFP screening, ultrasound examination, and possibly amniocentesis. Indications for use of PUBS include prenatal diagnosis of inherited blood disorders, karyotyping of malformed fetuses, detection of fetal infection, determination of the acid-base status of fetuses with intrauterine growth restriction, and assessment and treatment of isoimmunization and thrombocytopenia in the fetus. BPP is a method of assessing fetal well-being in the third trimester. Before amniocentesis is considered, the client first would have an ultrasound for direct visualization of the fetus.

A newborn is experiencing apnea and presents with microcephaly and seizures. Which would you expect to be included in the treatment plan? a. give one dose of rocephin to newborn at birth b. give newborn acyclovir and provide supportive measures c. no treatment necessary d. administer HepB immune globulin to the newborn

b. give newborn acyclovir and provide supportive measures

A 16 week pregnant patient comes in to the OB ED saying the baby is coming out. She delivers the fetus and is having moderate-heavy bleeding and severe cramping. What kind of miscarriage is this? a. inevitable miscarriage b. incomplete miscarriage c. threatened miscarriage d. complete miscarriage

b. incomplete miscarriage

A woman who's 36 weeks pregnant comes into the labor & delivery unit with mild contracts. Which of the following complications should the nurse watch out for when the client informs her that she has placenta prevue? a. sudden rupture of membranes b. vaginal bleeding c. emesis d. fever

b. vaginal bleeding contractions may disrupt the microvascular network in the placenta of a client with placenta prevue and result in bleeding. If the separation of the placenta occurs at the margin of the placenta, the blood will escape vaginally. Sudden rupture of the membranes isn't related to placenta prevue. Fever would indicate an infectious process, and emesis isn't related to placenta previa

A newborn is receiving phototherapy for the treatment of hyperbilirubinemia. The nurse evaluates that teaching has been effective when the parents demonstrate which behaviors?Select all that apply. a. Limit the infant's intake due to loose green stools. b. Cover the infant with a loose blanket while under the bililights. c. Continue breastfeeding during the jaundice. d. Cover the infant's eyes before placing him under the bililight. e. Keep the genitalia covered to prevent soiling.

c, d, e Explanation: The infant should be unclothed to allow as much skin exposure to the bililight as possible. Breastfeeding is not contraindicated with hyperbilirubinemia. Increased fluid intake will aid excretion of bilirubin and loose green stools are an indication that bilirubin is being excreted. It is important to protect the infant's eyes from the bililight to prevent permanent damage. It is acceptable practice to keep the genitalia covered to prevent soiling from urine or feces.

A father asks how the bilirubin lights make the newborn's bilirubin level go down. What is the best reply by the nurse? a. "Exposing the skin to the air helps get rid of the jaundice. The bililights really just keep the baby warm while this occurs." b. "The bililights release a substance in the body that attacks the bilirubin and destroys it." c. "The bililights help convert the bilirubin to a form the baby can get rid of." d. "The lights prevent more bilirubin from being released into your baby's body."

c. "The bililights help convert the bilirubin to a form the baby can get rid of." Explanation: Phototherapy assists the body in converting unconjugated bilirubin to conjugated bilirubin, which is water-soluble and easier for the body to eliminate.

The maternal-newborn nurse determines that which infant is at greatest risk for infection? a. 39 weeks' gestation, diagnosed with caput succedaneum b. 38 weeks' gestation, cesarean birth for breech presentation c. 38 weeks' gestation, small for gestational age (SGA) d. 41 weeks' gestation, infant of a diabetic mother (IDM)

c. 38 weeks' gestation, small for gestational age (SGA) Explanation: SGA infants often experience intrauterine growth restriction related to decreased blood flow to the placenta, which increases their risk for infection. In comparison, the infants in the other options are at less risk for infection. An infant born at 39 weeks' gestation, and diagnosed with caput succedaneum does not have an especially high risk for infection. An infant born at 38 weeks' gestation, by cesarean for breech presentation does not have an especially high risk for infection. An infant born at 41 weeks' gestation, from a diabetic mother (IDM) does not have an especially high risk for infection.

The following neonates are admitted to the nursery. The nurse should withhold the scheduled initial feeding on which newborn? a. A neonate with an axillary temperature of 36.4°C (97.5°F) b. A neonate who is small for gestational age (SGA) c. A neonate with a sustained respiratory rate of 68 breaths/min d. A neonate with a sustained heart rate of 118 beats/min

c. A neonate with a sustained respiratory rate of 68 breaths/min Explanation:A heart rate of 118 is slightly below the normal range of 120-160 beats/min, but it is not a contraindication to feeding the infant. A hypothermic infant is at risk for hypoglycemia, and requires a consistent source of glucose. Feeding a baby with a respiratory rate greater than 60 breaths/min orally increases the risk of aspiration. An infant who is SGA is at risk for hypoglycemia, and requires a consistent source of glucose.

A baby's mother is hepatitis B-positive. Which nursing interventions is most important when planning care for this newborn? a. Prepare for exchange transfusion. b. Isolate the newborn. c. Administer hepatitis B vaccine within 12 hours after delivery. d. Assess for HIV risk factors.

c. Administer hepatitis B vaccine within 12 hours after delivery. Explanation: Infants born to mothers who are hepatitis B-positive should receive a hepatitis B vaccine within 12 hours of birth to decrease their risk of acquiring the infection from maternal exposure. It is appropriate to assess for HIV risk factors in all infants, not just those at risk for hepatitis B. An exchange transfusion is not appropriate in this situation. Isolating the infant is not appropriate in this situation.

5. At 35 weeks of pregnancy a woman experiences preterm labor. Tocolytics are administered and she is placed on bed rest, but she continues to experience regular uterine contractions, and her cervix is beginning to dilate and efface. What would be an important test for fetal well-being at this time? a. Percutaneous umbilical blood sampling (PUBS) b. Ultrasound for fetal size c. Amniocentesis for fetal lung maturity d. Nonstress test (NST)

c. Amniocentesis for fetal lung maturity Amniocentesis would be performed to assess fetal lung maturity in the event of a preterm birth. Indications for PUBS include prenatal diagnosis or inherited blood disorders, karyotyping of malformed fetuses, detection of fetal infection, determination of the acid-base status of a fetus with intrauterine growth restriction, and assessment and treatment of isoimmunization and thrombocytopenia in the fetus. Typically, fetal size is determined by ultrasound during the second trimester and is not indicated in this scenario. NST measures the fetal response to fetal movement in a noncontracting mother.

A nurse is admitting the infant of a diabetic mother (IDM). At 1 hour of age, the nurse notices that the newborn is very jittery. Which action by the nurse is most appropriate? a. Begin oxygen by nasal cannula. b. Initiate use of a cardiac/apnea monitor. c. Assess the newborn's blood sugar. d. Place the newborn under a radiant warmer.

c. Assess the newborn's blood sugar. Explanation: The newborn is not showing any signs of hypoxia, so oxygen would not be appropriate. Infants of diabetic mothers are at risk for hypoglycemia after delivery. A primary sign of hypoglycemia is jitteriness. Putting the newborn under a warmer would not harm the infant, but is not the priority intervention at this time. Putting the newborn on a monitor would not harm the infant, but is not the priority intervention at this time.

The nurse is caring for a pregnant client with preeclampsia who is receiving a prescribed intravenous (IV) infusion of magnesium sulfate. To provide a safe environment, the nurse would ensure that which priority item is available? a. Reflex hammer b. Tongue blade c. Calcium gluconate injection d. Potassium chloride injection

c. Calcium gluconate injection Magnesium sulfate is a central nervous system depressant and relaxes smooth muscle. Toxic effects of magnesium sulfate may cause loss of deep tendon reflexes, heart block, respiratory paralysis, and cardiac arrest. The antidote for magnesium sulfate is calcium gluconate and should be available. An airway rather than a tongue blade is also an appropriate item. A percussion hammer may be important to assess reflexes but is not the priority item. Potassium chloride is not related to the administration of magnesium sulfate.

The nurse is caring for a client diagnosed with preeclampsia. When the client's condition progresses from preeclampsia to eclampsia, what would the nurse's first action be? a. Prepare to administer oxygen via face mask b. Administer an intravenous infusion of magnesium sulfate c. Prepare to maintain an open airway d. Assess the maternal blood pressure and fetal heart tones

c. Prepare to maintain an open airway Eclampsia is characterized by the occurrence of seizures. If the client experiences seizures, it is important as a first action to establish and maintain an open airway and prevent injuries to the client. The other options are all interventions that should be done but not initially.

Which is the most common complication for which the nurse must monitor preterm infants? a. Aspiration of mucus b. Hemorrhage c. Respiratory distress d. Brain damage

c. Respiratory distress Immaturity of the respiratory tract in preterm infants is evidenced by a lack of functional alveoli, smaller lumina with increased possibility of collapse of the respiratory passages, weakness of respiratory musculature, and insufficient calcification of the bony thorax, leading to respiratory distress. Hemorrhage is not a common occurrence at the time of birth unless trauma has occurred. Brain damage is not a primary concern unless severe hypoxia occurred during labor; it is difficult to diagnose at this time. Aspiration of mucus may be a problem, but generally the air passageway is suctioned as needed.

The nurse is teaching umbilical cord care to a new mother. What information would the nurse provide to the mother related to cord care? a. Cord care is done only at birth to control bleeding. b. Alcohol is the only agent to use to clean the cord. c. The process of keeping the cord clean and dry will decrease bacterial growth. d. It takes at least 21 days for the cord to dry up and fall off.

c. The process of keeping the cord clean and dry will decrease bacterial growth. The cord should be kept clean and dry to decrease bacterial growth. Usually the cord is cleansed with soap and water around base of the cord where it joins the skin. The primary health care provider is notified of any odor, discharge, or skin inflammation. The diaper should not cover the cord because a wet or soiled diaper will slow or prevent drying of the cord and foster infection. Cord care is required until the cord dries up and falls off between 7 and 14 days after birth.

A 20 day old baby presents to the pediatrician with signs of sepsis. The nurse confers the baby has most likely developed this infection from: a. maternal GBS positive b. chorioamnioitis c. maternal derived infection at home taking care of the infant d. maternal fever onset during labor

c. maternal derived infection at home taking care of the infant

The lab results show that a mother has a blood type of O positive and her infant has the blood type of A negative. As part of the plan of care the nurse should assess the infant for which condition a. breast milk jaundice b. physiologic hyperbilirubinemia c. pathologic hyperbilirubinemia d. Rh incompatibility

c. pathologic hyperbilirubinemia With the mother's blood type being O and the infant being type A, there is the potential for ABO incompatibility; antigen-antibody reactions are a cause of pathologic hyperbilirubinemia. For there to be Rh incompatibility the mother would need to be O negative, not O positive.

The nurse has conducted a class for pregnant clients diagnosed with diabetes mellitus about the signs/symptoms of potential complications. The nurse determines that the teaching was effective if a client makes which statement? a. "My insulin needs should decrease during the last 2 months because I will be using some of the baby's insulin supply." b. "I should not have ultrasounds done because I am diabetic." c. "I'm glad I don't have to worry about developing hypoglycemia while I am pregnant." d. "I need to watch my weight for any sudden gains because I could develop what they call gestational hypertension."

d. "I need to watch my weight for any sudden gains because I could develop what they call gestational hypertension." A diabetic pregnant client has a higher incidence of developing gestational hypertension than the nondiabetic pregnant client does. Ultrasounds are done frequently during a diabetic pregnancy to check for congenital anomalies and to determine appropriate growth patterns. Hypoglycemia is a problem during pregnancy in the client diagnosed with diabetes mellitus and needs to be assessed throughout the pregnancy. Insulin needs will increase during the last trimester because of increased hormone levels that destroy circulating insulin.

A postpartum client with a diagnosis of gestational diabetes is scheduled for discharge. During the discharge teaching, the client asks the nurse, "Do I have to worry about this diabetes anymore?" Which is the most appropriate response by the nurse? a. "You will have to worry about the diabetes only if you become pregnant again." b. "Your blood glucose level is within normal limits now, so you will be all right." c. "When you have gestational diabetes, you have diabetes forever, and you must be treated with medication for the rest of your life." d. "You will be at risk for developing gestational diabetes with your next pregnancy and also for developing diabetes mellitus."

d. "You will be at risk for developing gestational diabetes with your next pregnancy and also for developing diabetes mellitus." The client is at risk for developing gestational diabetes with each pregnancy. The client also has an increased risk for developing diabetes mellitus and needs to comply with follow-up assessments. She also needs to be taught techniques to lower her risk for developing diabetes mellitus, such as weight control. The diagnosis of gestational diabetes mellitus indicates that this client has an increased risk for developing diabetes mellitus; however, with proper care, it may not develop.

The nurse is caring for a 30-weeks'-gestation infant at risk for necrotizing enterocolitis (NEC). The nurse should observe for which of the following? a. Discolored feet b. A decrease in respiratory rate c. A bulging fontanelle d. Abdominal distention

d. Abdominal distention Explanation: A decreased respiratory rate should be reported to the health care provider, but is not related to NEC. Changes in the gastrointestinal assessment, including abdominal distention, occur with NEC. Discolored feet should be reported to the health care provider, but are not related to NEC. A bulging fontanelle should be reported to the health care provider, but is not related to NEC.

On admission to the nursery, it is noted that the mother's membranes were ruptured for 48 hours before delivery, and her temperature is 102°F. What information from this newborn's assessment should the nurse evaluate further? a. Excessive bruising of presenting part b. Jitteriness c. Irregular respiratory rate d. Axillary temperature 97.2°F

d. Axillary temperature 97.2°F Explanation: This newborn is at risk for sepsis caused by prolonged rupture of membranes and maternal fever. A primary sign of sepsis in the newborn is temperature instability, particularly hypothermia. An irregular respiratory pattern is normal. Jitteriness could be a sign of hypoglycemia. Excessive bruising is often related to a difficult delivery with an increased risk of hyperbilirubinemia.

1. A woman arrives at the clinic seeking confirmation that she is pregnant. The following information is obtained: She is 24 years old with a body mass index (BMI) of 17.5. She admits to having used cocaine "several times" during the past year and drinks alcohol occasionally. Her blood pressure (BP) is 108/70 mm Hg, her pulse rate is 72 beats/min, and her respiratory rate is 16 breaths/min. The family history is positive for diabetes mellitus and cancer. Her sister recently gave birth to an infant with a neural tube defect (NTD). Which characteristics place the woman in a high risk category? a. Blood pressure, age, BMI b. Drug/alcohol use, age, family history c. Family history, blood pressure, BMI d. Family history, BMI, drug/alcohol abuse

d. Family history, BMI, drug/alcohol abuse

The nurse can best promote parental bonding with a high-risk newborn being transferred to the neonatal intensive care unit (NICU) by doing which of the following? a. Encouraging the parents to call the NICU daily. b. Allowing parents to see the newborn for 15 minutes three times each day. c. Not discussing how sick the infant is. d. Giving the parents a picture of the baby prior to transport to the NICU.

d. Giving the parents a picture of the baby prior to transport to the NICU. Explanation: Parents should be given a picture of the infant before the baby is transported. Calling the unit to check on their baby might help bonding, but seeing the baby is more effective. Parents are typically allowed to visit as often and for as long as they want. It is important to be honest with parents, even if the prognosis is poor.

A client who received no prenatal care delivers a 9-pound, 4-ounce baby boy who exhibits signs of respiratory distress. The nurse obtains a blood sample from the infant per protocol to assess for which potential problem? a. Hyperbilirubinemia b. Sepsis c. Hemolysis d. Hypoglycemia

d. Hypoglycemia Explanation: The blood sample is not being drawn to assess for hemolysis. The blood sample is not being drawn to assess for hyperbilirubinemia. A 9-pound, 4-ounce infant is large for gestational age (LGA). An LGA infant who demonstrates respiratory distress could have a diabetic mother. The infant produces his own insulin during pregnancy, and stores the excess glucose as fat to compensate for high maternal glucose loads. After delivery the infant is at high risk for hypoglycemia because excess maternal glucose is now absent from the infant's circulation. The blood sample is not being drawn to assess for sepsis.

Which nursing diagnosis should be the highest priority of the nurse who is caring for a preterm newborn? a. Anticipatory Grieving related to loss of "perfect delivery" b. Risk for Injury related to thin epidermis c. Imbalanced Nutrition: Less than Body Requirements related to immature digestive system d. Ineffective Thermoregulation related to lack of subcutaneous fat

d. Ineffective Thermoregulation related to lack of subcutaneous fat Explanation: Newborns compensate for hypothermia by metabolizing brown fat. This process requires glucose and oxygen. Preterm newborns are at risk for hypoglycemia and respiratory distress, so hypoglycemia can further increase their needs for oxygen and glucose, and can cause serious complications. This nursing diagnosis is appropriate, but is not the highest priority because it relates to psychosocial needs. This nursing diagnosis is appropriate, but is not the highest priority.

Nurses should be aware of the strengths and limitations of various biochemical assessments during pregnancy, including that: a. Chorionic villus sampling (CVS) is becoming more popular because it provides early diagnosis. b. Maternal serum alpha-fetoprotein (MSAFP) screening is recommended only for women at risk for neural tube defects. c. Percutaneous umbilical blood sampling (PUBS) is one of the triple-marker tests for Down syndrome. d. MSAFP is a screening tool only; it identifies candidates for more definitive procedures.

d. MSAFP is a screening tool only; it identifies candidates for more definitive procedures.

A newborn's mother has a history of prenatal narcotic abuse. Which nursing interventions would be most appropriate for this infant?Select all that apply. a. Place a mobile on crib. b. Encourage family members to stroke and talk to the infant. c. Hold and rock the infant as much as possible. d. Offer the infant a pacifier. e. Position the infant on the right side or in semi-Fowler's position.

d. Offer the infant a pacifier. e. Position the infant on the right side or in semi-Fowler's position. Explanation: Holding and rocking involves increasing environmental stimulation. This is contraindicated in these infants because they are already hyperstimulated from the drug withdrawal process. Infants experiencing neonatal abstinence syndrome (NAS) often have an increased need for non-nutritive sucking, and offering a pacifier would help meet this need. A mobile increases environmental stimulation. This is contraindicated in these infants because they are already hyperstimulated from the drug withdrawal process. Talking and stroking the infant involve increasing environmental stimulation. This is contraindicated in these infants because they are already hyperstimulated from the drug withdrawal process. It is good to place the infant on the right side or in semi-Fowler's position to avoid possible aspiration of vomitus or secretions.

The nurse would take which action as part of nursing care of the baby experiencing neonatal abstinence syndrome? a. Spend extra time holding and rocking the baby. b. Place stuffed animals and mobiles in the crib to provide visual stimulation. c. Avoid the use of pacifiers. d. Position the baby's crib in a quiet corner of the nursery.

d. Position the baby's crib in a quiet corner of the nursery. Explanation: Neonatal abstinence syndrome, or drug withdrawal, causes hyperstimulation of the neonate's nervous system. Nursing interventions should focus on decreasing environmental and sensory stimulation during the withdrawal period.

A mother was diagnosed with gonorrhea immediately after delivery. When providing nursing care for the infant, what would be an important goal of the nurse? a. Prevent the development of thrush. b. Teach the danger of breastfeeding with gonorrhea. c. Lubricate the eyes. d. Prevent the development of ophthalmia neonatorum.

d. Prevent the development of ophthalmia neonatorum. Explanation: A newborn can become infected with gonorrhea as it passes through the birth canal. Gonorrhea can cause permanent blindness in the newborn, called ophthalmia neonatorum. All babies' eyes are treated with an antibiotic prophylactically after birth. The eyes require an antibiotic rather than a lubricant. Oral thrush is not a concern although eye care is a priority. Teaching does not take priority over physiological needs of the newborn or mother.

Which neonate requires the closest observation by the nurse? a. The neonate with enlarged breast tissue b. The baby who startles at loud sounds c. The neonate with irregular respirations at 30-40 breaths/minute d. The neonate whose color became cyanotic during the first feeding

d. The neonate whose color became cyanotic during the first feeding Explanation: Being startled at loud noises is normal for a neonate. Enlarged breast tissue in neonatesis normal for a neonate. Central cyanosis is always considered abnormal, and warrants further evaluation.

A primigravida is receiving magnesium sulfate for the treatment of gestational hypertension. The nurse who is caring for the client is performing assessments every 30 minutes. Which finding would be of most concern to the nurse? A. Urinary output of 20 mL B. Deep tendon reflexes of 2+ C. Fetal heart rate of 120 beats/minute D. Respiratory rate of 10 breaths/minute

D not A bc ABCs but also, we are assessing q 30 min so urine output of 20 mL/30 mins is actually good

A baby is born to a type 1 diabetic mother. Which of the following lab values would the nurse expect the neonate to exhibit? 1. Plasma glucose 30 mg/dL. 2. Red blood cell count 1 million/mm3. 3. White blood cell count 2,000/mm3. 4. Hemoglobin 8 g/dL.

1 The nurse should anticipate that the plasma glucose levels would be low.

A baby is in the NICU whose mother was addicted to heroin during the pregnancy. Which of the following nursing actions would be appropriate? 1. Tightly swaddle the baby. 2. Place the baby prone in the crib. 3. Provide needed stimulation to the baby. 4. Feed the baby half-strength formula.

1 Tightly swaddling drug-addicted babies often helps to control the hyperreflexia that they may exhibit.

An 18 week pregnant patient comes in with moderate bleeding, cramping and cervical dilation then this is considered which type of miscarriage? a. inevitable miscarriage b. incomplete miscarriage c. threatened miscarriage d. complete miscarriage

a. inevitable miscarriage

The newborn is experiencing Acute Bilirubin Encephalopathy, what signs and symptoms indicate the third phase? Select all that apply. a. poor suck b. apnea c. fever d. hypotonia e. shrill cry

b. apnea e. shrill cry

The nurse is teaching a pregnant pt w diabetes about nutrition & insulin needs during pregnancy. The nurse should provide the pt w which info? A. Glucose crosses the placenta B. Insulin crosses the placenta C. Increased caloric intake is needed D. Decreased caloric intake is required

A glucose definitely crosses that placenta! the whole reason that happens is to allow more glucose available for baby (mom & babies levels are proportionate). insulin doesn't cross placenta (baby makes its own insulin). pts w diabetes do not need more or less caloric intake during intake (their distribution of carbs may be diff but they need the same amount of calories)

A high-risk labor client progresses from preeclampsia to eclampsia. Aggressive management is instituted, and the fetus is delivered via cesarean section. Which finding in the immediate postoperative period indicates that the client is at risk of developing HELLP syndrome? a. Platelet count of 50,000/mL b. Liver enzyme levels within normal range c. Negative for edema d. No evidence of nausea or vomiting

A ~ HELLP syndrome is characterized by hemolysis, elevated liver enzyme levels, and a low platelet count. A platelet count of 50,000/mL indicates thrombocytopenia.

Intrauterine growth restriction (IUGR) is associated with numerous pregnancy-related risk factors (Select all that apply). a. Poor nutrition b. Maternal collagen disease c. Gestational hypertension d. Premature rupture of membranes e. Smoking

A, B, C, E

Transvaginal ultrasonography is often performed during the first trimester. While preparing your 6-week gestation patient for this procedure, she expresses concerns over the necessity for this test. The nurse should explain that this diagnostic test may be indicated for a number of situations (Select all that apply). a. Multifetal gestation b. Obesity c. Fetal abnormalities d. Amniotic fluid volume e. Ectopic pregnancy

A, B, C, E

Which characteristics would the nurse anticipate in the infant of a suspected or known opioid abuser? Select all that apply. a. Prolonged sleep periods b. Dehydration c. Hyperactivity d. Muscle hypotonicity e. Tremors

c. Hyperactivity e. Tremors Opioid dependence in the newborn is physiologic; as the drug is cleared from the body, signs of drug withdrawal become evident. Tremors and hyperactivity are typical signs of cerebral irritability. Dehydration is a result of inadequate feeding, not a direct result of opioid withdrawal. Muscle hypertonicity, not hypotonicity, occurs with opioid withdrawal. Signs of opioid withdrawal include excessive activity and sleep disturbances.

The nurse recognizes that a nonstress test (NST) in which two or more fetal heart rate (FHR) accelerations of 15 beats/min or more occur with fetal movement in a 20-minute period is: a. Nonreactive b. Positive c. Negative d. Reactive

d. Reactive

All infants born to mothers with diabetes are at some risk for complications. True or false?

True The degree of risk is influenced by the severity and duration of maternal disease.

Which is the only known cure for preeclampsia? a. Magnesium sulfate b. Delivery of the fetus c. Antihypertensive medications d. Administration of aspirin (ASA) every day of the pregnancy

B ~If the fetus is viable and near term, birth is the only known cure for preeclampsia. Magnesium sulfate is one of the medications used to treat but not cure preeclampsia. Antihypertensive medications are used to lower the dangerously elevated blood pressures in preeclampsia and eclampsia. Low doses of aspirin (60 to 80 mg) have been administered to women at high risk for developing preeclampsia.

The nurse finds the mother of a 28-weeks'-gestation infant crying in her room. The mother states, "I just know my baby is going to die." What is the most therapeutic response by the nurse? a. "My baby was born at 27 weeks, and he is fine now." b. "You seem very worried about what will happen to your baby." c. "I know this seems overly optimistic, but it is likely that everything will be fine." d. "Why do you think that?"

b. "You seem very worried about what will happen to your baby." Explanation: It is not therapeutic to give false reassurance to the client. It is not therapeutic to ask clients "why" they feel the way they do. Reflecting on what the client said offers her an opportunity to share her feelings. A therapeutic response focuses on the client rather than the nurse.

A nonstress test is performed on a client, and the results are documented in the chart as no accelerations during a 40-minute observation. The nurse interprets these findings as which result? A. A nonreactive nonstress test B. Equivocal C. A reactive nonstress test D. Unsatisfactory

A

The nurse is planning care for a newborn of a mother with diabetes mellitus. What is the priority nursing consideration for this newborn? 1. Developmental delays because of excessive size 2. Maintaining safety because of low blood glucose levels 3. Choking because of impaired suck and swallow reflexes 4. Elevated body temperature because of excess fat and glycogen

2 The newborn of a diabetic mother is at risk for hypoglycemia, so maintaining safety because of low blood glucose levels would be a priority. The newborn would also be at risk for hyperbilirubinemia, respiratory distress, hypocalcemia, and congenital anomalies. Developmental delays, choking, and an elevated body temperature are not expected problems.

A neonate is being assessed for necrotizing enterocolitis (NEC). Which of the following actions by the nurse is appropriate? (Select all that apply) 1. Perform hemoccult test on stools. 2. Monitor for an increase in abdominal girth. 3. Measure gastric contents before each feed. 4. Assess bowel sounds before each feed. 5. Assess for anal fissures daily.

1, 2, 3, 4 1. Babies with necrotizing enterocolitis (NEC) have blood in their stools. 2. The abdominal girth measurements of babies with NEC increase. 3. When babies have NEC, they have increasingly larger undigested gastric contents after feeds. 4. The neonates' bowel sounds are diminished with NEC. (5. The presence of anal fissures is unrelated to NEC.)

The nurse is caring for a pt who has just delivered a newborn following a pregnancy with placenta previa. When reviewing the plan of care, the nurse should prepare to monitor the pt for which risk that is associated with placenta previa? A. Hematoma B. Hemorrhage C. Chronic hypertension D. Disseminated intravascular coagulation

B

The nurse prepares a plan of care for the pt w preeclampsia & documents that if the pt progresses from preeclampsia to eclampsia, the nurse should take which first action? A. Administer oxygen by face mask. B. Clear and maintain an open airway. C. Administer magnesium sulfate intravenously. D. Assess the blood pressure and fetal heart rate.

B

Which analysis of maternal serum may predict chromosomal abnormalities in the fetus? a. Multiple-marker screening b. Lecithin/sphingomyelin (L/S) ratio c. Biophysical profile d. Type and crossmatch of maternal and fetal serum

a. Multiple-marker screening

The nurse is caring for a primigravida mother who tested positive for Tay-Sachs on a quad screen. What can the nurse do to help? a. give resources for comfort care b. provide time for grieving c. dismiss hopeful questions d. reassure the mother that everything will be okay e. prepare the mother for the death of the child f. involve the family in the plan of care

a. give resources for comfort care b. provide time for grieving e. prepare the mother for the death of the child f. involve the family in the plan of care

Risk factors tend to be interrelated and cumulative in their effect. While planning the care for a laboring client with diabetes mellitus, the nurse is aware that she is at a greater risk for: a. Oligohydramnios. b. Polyhydramnios. c. Postterm pregnancy. d. Chromosomal abnormalities.

b. Polyhydramnios.

Upon delivery an infant presents with rhonchi, APGAR score of 4 and 5. The nurse notices meconium stains on the infants skin. What should be the nurses PRIORITY action? a. vigorously dry the infant b. provide oxygen c. place the infant in an incubator d. suction the mouth

d. suction the mouth

A 1-day-old neonate, 32 weeks' gestation, is in an overhead warmer. The nurse assesses the morning axillary temperature as 96.9°F. Which of the following could explain this assessment finding? 1. This is a normal temperature for a preterm neonate. 2. Axillary temperatures are not valid for preterm babies. 3. The supply of brown adipose tissue is incomplete. 4. Conduction heat loss is pronounced in the baby.

3

A nurse suspects that a pregnant client may be experiencing abruption placenta based on assessment of which of the following? (Select all that apply.) A) Dark red vaginal bleeding B) Insidious onset C) Absence of pain D) Rigid uterus E) Absent fetal heart tones

Ans: A, D, E Feedback: Assessment findings associated with abruption placenta include a sudden onset, with concealed or visible bleeding, dark red bleeding, constant pain or uterine tenderness on palpation, firm to rigid uterine tone, and fetal distress or absent fetal heart tones.

A woman is receiving magnesium sulfate as part of her treatment for severe preeclampsia. The nurse is monitoring the woman's serum magnesium levels. Which level would the nurse identify as therapeutic? A) 3.3 mEq/L B) 6.1 mEq/L C) 8.4 mEq/L D) 10.8 mEq/L

Ans: B Although exact levels may vary among agencies, serum magnesium levels ranging from 4 to 7 mEq/L are considered therapeutic, whereas levels more than 8 mEq/dL are generally considered toxic.

An abortion when the fetus dies but is retained in the uterus is called: a. inevitable. b. missed. c. incomplete. d. threatened.

B ~ A missed abortion refers to a dead fetus being retained in the uterus. An inevitable abortion means that the cervix is dilating with the contractions. An incomplete abortion means that not all the products of conception were expelled. With a threatened abortion, the client has cramping and bleeding but not cervical dilation.

A pregnant woman's biophysical profile score is 8. She asks the nurse to explain the results. The nurse's best response is: a. "The test results are within normal limits." b. "Immediate delivery by cesarean birth is being considered." c. "Further testing will be performed to determine the meaning of this score." d. "An obstetric specialist will evaluate the results of this profile and, within the next week, will inform you of your options regarding delivery."

a. "The test results are within normal limits."

A woman is undergoing a nipple-stimulated contraction stress test (CST). She is having contractions that occur every 3 minutes. The fetal heart rate (FHR) has a baseline of approximately 120 beats/min without any decelerations. The interpretation of this test is said to be: a. Negative. b. Positive. c. Satisfactory. d. Unsatisfactory.

a. Negative.

Which of the following signs would indicate that a newborn with hyperbilirubinemia is worsening? Select all that apply a. hypotonia b. positive Babinski reflex c. respiratory rate of 50 d. bilirubin level greater than 25 e. yellow discoloration of the skin

a. hypotonia d. bilirubin level greater than 25

A 42-week gravida is delivering her baby. A nurse and pediatrician are present at the birth. The amniotic fluid is green and thick. The baby fails to breathe spontaneously. Which of the following actions should the nurse take next? 1. Stimulate the baby to breathe. 2. Assess neonatal heart rate. 3. Assist with intubation. 4. Place the baby in the prone position.

3 Before breathing, the baby must be intubated so that the meconiumcontaminated fluid can be aspirated from the baby's airway.

The abuse of which of the following substances during pregnancy is the leading cause of mental retardation in the United States? A. Alcohol B. Tobacco C. Marijuana D. Heroin

A A. Correct: Alcohol abuse during pregnancy is recognized as one of the leading causes of mental retardation in the United States. B. Incorrect: Alcohol abuse during pregnancy is recognized as one of the leading causes of mental retardation in the United States. C. Incorrect: Alcohol abuse during pregnancy is recognized as one of the leading causes of mental retardation in the United States. D. Incorrect: Alcohol abuse during pregnancy is recognized as one of the leading causes of mental retardation in the United States.

What history would lead you to suspect an ectopic pregnancy in a client at 8 weeks gestation presenting with abdominal pain and bleeding? a. Treated 1 year ago for pelvic inflammatory disease (PID) b. Oral contraception for last 3 years c. Urinary frequency for 1 week d. Irregular cycles for 1 year prior to conception

A ~ PID causes fallopian tube damage. Blockage of the tube prevents movement of the fertilized ovum, resulting in implantation in the tube. Oral contraception for the last 3 years, urinary frequency for 1 week, and irregular cycles for 1 year prior to conception have no effect on the development of ectopic pregnancy.

Which strategy should the nurse anticipate implementing during antepartum management of a pregnant client who is hospitalized at 33 weeks of gestation for preeclampsia with severe​ manifestations? (Select all that​ apply.) A. Ongoing assessment of the need for prompt delivery B. Steroid administration C. Fetal surveillance D. Administration of magnesium sulfate E. Activity and dietary restrictions

A,B,C,D Rationale: Fetal​ surveillance, steroid administration to accelerate fetal lung​ maturity, administration of magnesium sulfate prophylactically to prevent​ seizures, and ongoing assessment of the need for prompt delivery are all interventions that may be used during antepartum management of a client with preeclampsia. Activity and dietary restrictions do not alter the course or outcome of​ preeclampsia, so the nurse would not be implementing strategies related to this.

A client with preeclampsia at 32 weeks of gestation has been admitted to the hospital with signs of a worsening condition. She tells the nurse that she is worried about injury to her baby. Which action should the nurse take to help the client remain calm about her own and her​ baby's condition?​ (Select all that​ apply.) A. Educating the client on how to monitor and record fetal movement throughout the day B. Inviting the client to identify and discuss any concerns she has about her​ baby's well-being C. Informing the client that a preterm delivery may be unavoidable if she does not remain calm and her blood pressure continues to rise D. Keeping the client and her family informed about fetal status E. Informing the client that a nurse will be with her to offer support during the administration of any tests for fetal​ well-being

A,B,D,E A client with preeclampsia at 32 weeks of gestation has been admitted to the hospital with signs of a worsening condition. She tells the nurse that she is worried about injury to her baby. Which action should the nurse take to help the client remain calm about her own and her​ baby's condition?​ (Select all that​ apply.) A. Educating the client on how to monitor and record fetal movement throughout the day B. Inviting the client to identify and discuss any concerns she has about her​ baby's well-being C. Informing the client that a preterm delivery may be unavoidable if she does not remain calm and her blood pressure continues to rise D. Keeping the client and her family informed about fetal status E. Informing the client that a nurse will be with her to offer support during the administration of any tests for fetal​ well-being

Which clinical findings would alert the nurse that the neonate is expressing pain? a.Low-pitched crying; tachycardia; eyelids open wide b.Cry face; flaccid limbs; closed mouth c.High-pitched, shrill cry; withdrawal; change in heart rate d.Cry face; eyes squeezed; increase in blood pressure

ANS: D Crying and an increased heart rate are manifestations indicative of pain in the neonate. Typically, infants tightly close their eyes when in pain, not open them wide. In addition, infants may display a rigid posture with the mouth open and may also withdraw limbs and become tachycardic with pain. A high-pitched, shrill cry is associated with genetic or neurologic anomalies.

After reviewing a client's history, which factor would the nurse identify as placing her at risk for gestational hypertension? A) Mother had gestational hypertension during pregnancy. B) Client has a twin sister. C) Sister-in-law had gestational hypertension. D) This is the client's second pregnancy.

Ans: A A family history of gestational hypertension, such as a mother or sister, is considered a risk factor for the client. Having a twin sister or having a sister-in-law with gestational hypertension would not increase the client's risk. If the client had a history of preeclampsia in her first pregnancy, then she would be at risk in her second pregnancy.

The priority nursing intervention when admitting a pregnant client who has experienced a bleeding episode in late pregnancy is to: a. monitor uterine contractions. b. assess fetal heart rate and maternal vital signs. c. place clean disposable pads to collect any drainage. d. perform a venipuncture for hemoglobin and hematocrit levels.

B ~ Assessment of the fetal heart rate (FHR) and maternal vital signs will assist the nurse in determining the degree of the blood loss and its effect on the client and fetus. Monitoring uterine contractions is important, but not the top priority. It is important to assess future bleeding, but the top priority is client and fetal well-being. The most important assessment is to check client and fetal well-being. The blood levels can be obtained later.

A client taking magnesium sulfate has a respiratory rate of 10 breaths/min. In addition to discontinuing the medication, which action should the nurse take? a. Increase the clients IV fluids. b. Administer calcium gluconate. c. Vigorously stimulate the client. d. Instruct the client to take deep breaths.

B ~ Calcium gluconate reverses the effects of magnesium sulfate. Increasing the clients IV fluids will not reverse the effects of the medication. Stimulation will not increase the respirations. Deep breaths will not be successful in reversing the effects of the magnesium sulfate.

The nurse is monitoring a client with severe preeclampsia who is on IV magnesium sulfate. Which signs of magnesium toxicity should the nurse monitor for? (SATA) a. Cool, clammy skin b. Altered sensorium c. Pulse oximeter reading of 95% d. Respiratory rate of less than 12 breaths/min e. Absence of deep tendon reflexes

B, D, E ~ Signs of magnesium toxicity include the following: Respiratory rate of less than 12 breaths/min (hospitals may specify a rate < 14 breaths/min) Maternal pulse oximeter reading lower than 95% Absence of deep tendon reflexes Sweating, flushing Altered sensorium (confused, lethargic, slurred speech, drowsy, disoriented) Hypotension Serum magnesium value above the therapeutic range of 4 to 8 mg/dL Cold, clammy skin and a pulse oximeter reading of 95% would not be signs of toxicity.

Which intervention should the nurse implement to address clinical manifestations due to the increased vascular permeability for a client with​ preeclampsia? (Select all that​ apply.) A. Providing regular antenatal fetal​ surveillance/continuous intrapartum fetal monitoring B. Monitoring breath sounds and oxygen saturation C. Elevating extremities to reduce edema D. Administering an anticonvulsant and magnesium sulfate E. Reducing external stimuli

B,C ​Rationale: To address clinical manifestations due to the increased vascular permeability of​ preeclampsia, the nurse would elevate extremities and monitor breath sounds and oxygen saturation. Reduction of external stimuli and administration of magnesium sulfate would address cerebral edema and vasospasm. Regular antenatal fetal​ surveillance/continuous intrapartum fetal monitoring would address loss of normal vasodilation of uterine arterioles.

Which nursing intervention is appropriate to include in the plan of care for the client with gestational​ hypertension? (Select all that​ apply.) A. Educating the client about treatment alternatives for an ectopic pregnancy B. Considering cultural limitations when educating the client about nutritious meal planning C. Taking frequent blood pressure readings D. Assessing the​ client's blood glucose level once daily before breakfast E. Educating the client about the effect of the disease process on pregnancy

B,C,E Rationale: Assessing blood pressure more​ frequently, educating the client about proper​ nutrition, and helping the client maintain her physical and emotional comfort are important nursing interventions. It is important to take cultural considerations into account during the educational process to help ensure compliance. The client with gestational hypertension has no need for information about ectopic pregnancy. Blood glucose levels do not need to be monitored in a woman with gestational hypertension.

The prenatal nurse is reviewing the histories of several clients recently confirmed as pregnant. Which client should the nurse identify as having a high risk for​ preeclampsia? (Select all that​ apply.) A. Hispanic decent B. History of kidney disease C. Body mass index 30.1 D. Maternal age of 32 E. Twin pregnancy

B,C,E Rationale: Some predisposing risk factors for preeclampsia are maternal age of 40 years or​ older, obesity, medical history of chronic hypertension or kidney​ disease, previous​ preeclampsia/eclampsia, presence of​ multiples, and being of African descent.

With regard to amniocentesis, nurses should be aware that: a. Because of new imaging techniques, amniocentesis is now possible in the first trimester. b. Despite the use of ultrasound, complications still occur in the mother or infant in 5% to 10% of cases. c. Administration RhoD immunoglobulin may be necessary. d. The presence of meconium in the amniotic fluid is always cause for concern.

c. Administration RhoD immunoglobulin may be necessary.

all fetal tissue passed, cervix closed, mild cramping, small amount of bleeding. what type of miscarriage is this? a. inevitable miscarriage b. incomplete miscarriage c. threatened miscarriage d. complete miscarriage

d. complete miscarriage

a pregnant pt came in to get a contraction stress test. we put her on the monitor & they are having contractions for 20 minutes & have a total of 6 contractions. she had late decels w 4 of the contractions. Would this be a negative or positive test result?

positive

The nurse in a newborn nursery is monitoring a preterm newborn for respiratory distress syndrome. Which assessment findings would alert the nurse to the possibility of this syndrome? 1. Tachypnea and retractions 2. Acrocyanosis and grunting 3. Hypotension and bradycardia 4. Presence of a barrel chest and acrocyanosis

1 A newborn infant with respiratory distress syndrome may present with clinical signs of cyanosis, tachypnea or apnea, nasal flaring, chest wall retractions, or audible grunts. Acrocyanosis, a bluish discoloration of the hands and feet, is associated with immature peripheral circulation, and is common in the first few hours of life. Options 2, 3, and 4 do not indicate clinical signs of respiratory distress syndrome.

A neonate is found to have choanal atresia on admission to the nursery. Which of the following physiological actions will be hampered by this diagnosis? 1. Feeding. 2. Digestion. 3. Immune response. 4. Glomerular filtration.

1 Choanal atresia will impact the baby's ability to feed.

Based on maternal history of alcohol addiction, a baby in the neonatal nursery is being monitored for signs of fetal alcohol syndrome (FAS). The nurse should assess this baby for which of the following? 1. Poor suck reflex. 2. Ambiguous genitalia. 3. Webbed neck. 4. Absent Moro reflex.

1 FAS babies usually have a very weak suck.

The nurse in the newborn nursery is performing admission vital signs on a newborn infant. The nurse notes that the respiratory rate of the newborn is 50 breaths per minute. Which action should the nurse take? 1. Document the findings. 2. Contact the health care provider. 3. Apply an oxygen mask to the newborn infant. 4. Cover the newborn infant with blankets and reassess the respiratory rate in 15 minutes.

1 The normal respiratory rate for a normal newborn is 30 to 60 breaths per minute. On assessment, if the nurse noted a respiratory rate of 50 breaths per minute, the nurse should document these findings because they are normal. Options 2, 3, and 4 are inappropriate or unnecessary nursing actions.

A baby was born 24 hours ago to a mother who received no prenatal care. The infant has tremors, sneezes excessively, constantly mouths for food, and has a shrill, highpitched cry. The baby's serum glucose levels are normal. For which of the following should the nurse request an order from the pediatrician? 1. Urine drug toxicology test. 2. Biophysical profile test. 3. Chest and abdominal ultrasound evaluations. 4. Oxygen saturation and blood gas assessments.

1 The symptoms are characteristic of neonatal abstinence syndrome. A urine toxicology would provide evidence of drug exposure.

A 42-week-gestation baby, 2,400 grams, whose mother had no prenatal care, is admitted into the NICU. The neonatologist orders blood work. Which of the following laboratory findings would the nurse expect to see? 1. Blood glucose 30 mg/dL. 2. Leukocyte count 1,000 cells/mm3. 3. Hematocrit 30%. 4. Serum pH 7.8.

1 This baby is small-for-gestational age. Full-term babies (40 weeks' gestation) should weigh between 2500 and 4000 grams. It is very likely that this baby used up his glycogen stores in utero because of an aging placenta. An aging placenta is unable to deliver sufficient nutrients to the fetus. As a result the fetus must use its glycogen stores to sustain life and, therefore, is high risk for hypoglycemia after birth.

A baby exhibits weak rooting and sucking reflexes. Which of the following nursing diagnoses would be appropriate? 1. Risk for deficient fluid volume. 2. Activity intolerance. 3. Risk for aspiration. 4. Feeding self-care deficit.

1 When a baby roots and sucks poorly, the baby is unable to transfer milk effectively. Since milk intake is the baby's source of fluid, the baby is high risk for fluid volume deficit. (not 4 bc babies are incapable of self-care.)

a client is being admitted to the antepartum unit for hypovolemia secondary to hyperemesis gravidarum. WHich of the following factors predisposes a client to the development of this? 1. trophoblastic disease 2. maternal age > 35 y.o. 3. malnourished or underweight clients 4. low levels of HCG

1. Trophoblastic disease is associated w/ hyperemesis grav obesity and maternal age younger than 20 y.o. are risk factors too. High levels of estrogen HCG have also been associated with the development.

The nursery room nurse is assessing a newborn infant who was born to a mother who abuses alcohol. Which assessment finding should the nurse expect to note? 1. Lethargy 2. Irritability 3. Higher-than-normal birth weight 4. A greater-than-normal appetite when feeding

2 Characteristic behaviors of the newborn infant with fetal alcohol syndrome (FAS) are similar to those of the drug-exposed newborn infant. These behaviors include irritability, tremors, poor feeding, and hypersensitivity to stimuli. Newborn infants with FAS are smaller at birth and present with failure to thrive. Head circumference and weight are most affected (smaller head circumference and decreased weight).

A 42-week-gestation baby has been admitted to the neonatal intensive care unit. At delivery, thick green amniotic fluid was noted. Which of the following actions by the nurse is critical at this time? 1. Bath to remove meconium-contaminated fluid from the skin. 2. Ophthalmic assessment to check for conjunctival irritation. 3. Rectal temperature to assess for septic hyperthermia. 4. Respiratory evaluation to monitor for respiratory distress.

4 Meconium aspiration syndrome (MAS) is a serious complication seen in postterm neonates who are exposed to meconium-stained fluid. Respiratory distress would indicate that the baby has likely developed MAS.

A pregnant woman presents in labor at term, having had no prenatal care. After birth, her infant is noted to be small for gestational age with small eyes and a thin upper lip. The infant also is microcephalic. Based on her infant's physical findings, this woman should be questioned about her use of which substance during pregnancy? A. Alcohol B. Cocaine C. Heroin D. Marijuana

A A. Correct: The description of the infant suggests fetal alcohol syndrome, which is consistent with maternal alcohol consumption during pregnancy. B. Incorrect: Fetal brain, kidney, and urogenital system malformations have been associated with maternal cocaine ingestions. C. Incorrect: Heroin use in pregnancy frequently results in IUGR. The infant may have a shrill cry and sleep cycle disturbances and may present with poor feeding, tachypnea, vomiting, diarrhea, hypothermia or hyperthermia, and sweating. D. Incorrect: Studies have found a higher incidence of meconium staining in infants born of mothers who used marijuana during pregnancy.

Which of the following would the nurse have readily available for a client who is receiving magnesium sulfate to treat severe preeclampsia? A) Calcium gluconate B) Potassium chloride C) Ferrous sulfate D) Calcium carbonate

Ans: A The antidote for magnesium sulfate is calcium gluconate, and this should be readily available in case the woman has signs and symptoms of magnesium toxicity.

With regard to central nervous system injuries to the infant during labor and birth, nurses should be aware that: A. Intracranial hemorrhage (ICH) as a result of birth trauma is more likely to occur in the preterm, low-birth-weight infant. B. Subarachnoid hemorrhage (the most common form of ICH) occurs in term infants as a result of hypoxia. C. In many infants, signs of hemorrhage in a full-term infant are absent and diagnosed only through laboratory tests. D. Spinal cord injuries almost always result from forceps-assisted deliveries.

C A. Incorrect: ICH as a result of birth trauma is more likely to occur in the full-term, large infant. B. Incorrect: Subarachnoid hemorrhage in term infants is a result of trauma; in preterm infants, it is a result of hypoxia. C. Correct: Abnormalities in lumbar punctures or red blood cell counts, for instance, or in visuals on CT scan might reveal a hemorrhage. D. Incorrect: Spinal cord injuries are almost always from breech births; they are rare today because cesarean birth often is used for breech presentation.

A plan of care for an infant experiencing symptoms of drug withdrawal should include: A. Administering chloral hydrate for sedation B. Feeding every 4 to 6 hours to allow extra rest C. Swaddling the infant snugly and holding the baby tightly D. Playing soft music during feeding

C A. Incorrect: Phenobarbital or diazepam may be administered to decrease CNS irritability. B. Incorrect: The infant should be fed in small, frequent amounts and burped well to diminish aspiration and maintain hydration. C. Correct: The infant should be wrapped snugly to reduce self-stimulation behaviors and protect the skin from abrasions. D. Incorrect: The infant should not be stimulated (such as with music), because this will increase activity and potentially increase CNS irritability.

The nurse is counseling a pregnant woman diagnosed with gestational diabetes at 29 weeks' gestation. Which information should the nurse discuss with the client? Select all that apply. A. Plan induction at 35 weeks. B. Plan amniocentesis at this time. C. Schedule a biophysical profile immediately. D. Plan for weekly non stress tests at 32 weeks. E. Obtain nutritional counseling with a dietitian.

D & E no need to schedule amnio at this time bc pt is 29 weeks and we have no indication that she needs further testing RIGHT NOW. usually we start testing at 32 weeks.

A nonstress test is performed on a pt who is pregnant, & the results of the test indicate nonreactive findings. The primary health care provider prescribes a contraction stress test, & the results are documented as negative. How should the nurse document this finding? A. A normal test result B. An abnormal test result C. A high risk for fetal demise D. The need for a cesarean section

a

A 6-month-old child developed kernicterus immediately after birth. Which of the following tests should be done to determine whether or not this child has developed any sequelae to the illness? a. Blood urea nitrogen and serum creatinine. b. Alkaline phosphatase and bilirubin. c. Hearing testing and vision assessment. d. Peak expiratory flow and blood gas assessments.

C. Because the central nervous system (CNS) may have been damaged by the high bilirubin levels, testing of the senses as well as motor and cognitive assessments are appropriate.

a 21 y.o. client, 6 weeks pregnant, is diagnosed with hyperemesis gravidum. This excessive vomiting during pregnancy will often result in which of the following? 1. bowel perforation 2. electrolyte imbalance 3. miscarriage 4. PIH

2. electrolyte imbalance Excessive vomiting in clients with hyperemesis grav often causes weight loss / fluid and electrolyte, acid base imbalance.

During a health maintenance visit at the pediatrician's office, the nurse notes that a breastfeeding baby has thrush. Which of the following actions should the nurse take? 1. Nothing because thrush is a benign problem. 2. Advise the mother to bottle feed until the thrush is cured. 3. Obtain an order for antifungals for both mother and baby. 4. Assess for other evidence of immunosuppression.

3

The nurse is assessing a newborn who was born to a mother who is addicted to drugs. Which assessment finding would the nurse expect to note during the assessment of this newborn? 1. Lethargy 2. Sleepiness 3. Constant crying 4. Cuddles when being held

3 A newborn of a woman using drugs is irritable. The infant is overloaded easily by sensory stimulation. The infant may cry incessantly and be difficult to console. The infant would hyperextend and posture rather than cuddle when being held.

Which of the following would lead the nurse to suspect cold stress syndrome in a newborn with a temperature of 96.5°F? 1. Blood glucose of 50 mg/dL. 2. Acrocyanosis. 3. Tachypnea. 4. Oxygen saturation of 96%

3 Babies who have cold stress syndrome will develop respiratory distress. One symptom of the distress is tachypnea.

69. A gravid woman, who is 42 weeks' gestation, has just had a 20-minute nonstress test (NST). Which of the following results would the nurse interpret as a reactive test? 1. Moderate fetal heart baseline variability. 2. Maternal heart rate accelerations to 140 bpm lasting at least 20 seconds. 3. Two fetal heart accelerations of 15 bpm lasting at least 15 seconds. 4. Absence of maternal premature ventricular contractions.

3. Two fetal heart accelerations of 15 bpm lasting at least 15 seconds. This is the definition of a reactive nonstress test—there are two fetal heart accelerations of 15 bpm lasting 15 or more seconds during a 20-minute period.

What would you expect a baby of a diabetic mother to have?

-macrosomia -hypoglycemia -cardiomyopathy -polycythemia & hyperbilirubinemia -respiratory distress syndrome

An initial assessment on a large-for-gestational age (LGA) newborn infant is being done. Which physical assessment technique should the nurse assist in performing to assess for evidence of birth trauma? 1. Palpate the clavicles for a fracture. 2. Auscultate the heart for a cardiac defect. 3. Blanch the skin for evidence of jaundice. 4. Perform Ortolani's maneuver for hip dislocation.

1 Because of the newborn infant's large size, there is an increased risk for shoulder dystocia. This may result in fractured clavicles or brachial plexus palsy or both. Other complications related to birth trauma include facial paralysis, phrenic nerve palsy, depressed skull fractures, hematomas, and bleeding. A cardiac defect would not be related to birth trauma, even though there is an increase in cardiac defects such as transposition of the great vessels in the LGA newborn infant. Jaundice would not be present initially. Hip dislocation is a congenital disorder and is not caused by birth trauma.

The nurse in the newborn nursery is assessing a neonate who was born of a mother addicted to cocaine. Which would the nurse expect to note in the neonate? 1. Tremors 2. Bradycardia 3. Flaccid muscles 4. Extreme lethargy

1 Clinical symptoms at birth in neonates exposed to cocaine in utero include tremors, tachycardia, marked irritability, muscular rigidity, hypertension, and exaggerated startle reflex. These infants are difficult to console and exhibit an inability to respond to voices or environmental stimuli. They are often poor feeders and have episodes of diarrhea.

A baby was just born to a mother who had positive vaginal cultures for group B streptococcus. The mother was admitted to the labor room 2 hours before the birth. For which of the following should the nursery nurse closely observe this baby? 1. Hypothermia. 2. Mottling. 3. Omphalocele. 4. Stomatitis.

1 Hypothermia in a neonate may be indicative of sepsis. (2. Mottling is commonly seen in neonates. It is considered a normal finding. 3. Oomphalocele is not related to group B strep exposure. 4. Stomatitis is not a sign associated with group B strep exposure.)

24. An infant admitted to the newborn nursery has a blood glucose level of 35 mg/dL. The nurse should monitor this baby carefully for which of the following? 1. Jaundice. 2. Jitters. 3. Erythema toxicum. 4. Subconjunctival hemorrhages.

2 Babies who are hypoglycemic will often develop jitters (tremors). (1. Jaundice is not related to blood glucose levels. 3. Erythema toxicum is the newborn rash. It is unrelated to blood glucose levels. 4. Subconjunctival hemorrhages are often evident in neonates. They are related to the trauma of delivery, not to blood glucose levels.)

The birth of a baby, weight 4,500 grams, was complicated by shoulder dystocia. Which of the following neonatal complications should the nursery nurse observe for? 1. Leg deformities. 2. Brachial palsy. 3. Fractured radius. 4. Buccal abrasions.

2 During a difficult delivery with shoulder dystocia, the brachial nerve can become stretched and may even be severed. The nurse should, therefore, observe the baby for signs of palsy.

A nurse in the newborn nursery suspects that a new admission, 42 weeks' gestation, was exposed to meconium in utero. What would lead the nurse to suspect this? 1. The baby is bradycardic. 2. The baby's umbilical cord is green. 3. The baby's anterior fontanel is sunken. 4. The baby is desquamating.

2. The baby's umbilical cord is green.

A baby is born with a diaphragmatic hernia. Which of the following signs/symptoms would the nurse observe in the delivery room? 1. Projectile vomiting. 2. High-pitched crying. 3. Respiratory distress. 4. Fecal incontinence.

3 The baby will develop respiratory distress very shortly after delivery. Abdominal organs are displaced into the thoracic cavity when a baby is born with a diaphragmatic hernia. Because of the defect, the respiratory tree does not develop completely. The newly delivered baby, therefore, is unable to breathe effectively.

The nurse is caring for a post-term, small-for-gestational age (SGA) newborn infant immediately after admission to the nursery. What should the nurse monitor as the priority? 1. Urinary output 2. Total bilirubin levels 3. Blood glucose levels 4. Hemoglobin and hematocrit levels

3 The most common metabolic complication in the SGA newborn infant is hypoglycemia, which can produce central nervous system abnormalities and mental retardation if not corrected immediately. Urinary output, although important, is not the highest-priority action, because the post-term SGA infant is typically dehydrated as a result of placental dysfunction. Hemoglobin and hematocrit levels are monitored because the post-term SGA infant exhibits polycythemia, although this also does not require immediate attention. The polycythemia contributes to increased bilirubin levels, usually beginning on the second day after delivery.

Which of the following neonates is at highest risk for cold stress syndrome? 1. Infant of diabetic mother. 2. Infant with Rh incompatibility. 3. Postdates neonate. 4. Down syndrome neonate.

3 Postdates babies are at high risk for cold stress syndrome because while still in utero they often metabolize the brown adipose tissue for nourishment when the placental function deteriorates.

The nurse is preparing a plan of care for a newborn with fetal alcohol syndrome. The nurse should include which priority intervention in the plan of care? 1. Allow the newborn to establish own sleep-rest pattern. 2. Maintain the newborn in a brightly lighted area of the nursery. 3. Encourage frequent handling of the newborn by staff and parents. 4. Monitor the newborn's response to feedings and weight gain pattern.

4 Fetal alcohol syndrome is caused by maternal alcohol use during pregnancy. A primary nursing goal for the newborn diagnosed with fetal alcohol syndrome is to establish nutritional balance after delivery. These newborns may exhibit hyperirritability, vomiting, diarrhea, or an uncoordinated sucking and swallowing ability. A quiet environment with minimal stimuli and handling would help establish appropriate sleep-rest cycles in the newborn as well. Options 1, 2, and 3 are inappropriate interventions.

A macrosomic infant of a non-insulin dependent diabetic mother has been admitted to the neonatal nursery. The baby's glucose level on admission to the nursery is 25 mg/dL and after a feeding of mother's expressed breast milk is 35 mg/dL. Which of the following actions should the nurse take at this time? 1. Nothing, because the glucose level is normal for an infant of a diabetic mother. 2. Administer intravenous glucagon slowly over five minutes. 3. Feed the baby a bottle of dextrose and water and reassess the glucose level. 4. Notify the neonatologist of the abnormal glucose levels.

4 If the glucose level has not risen to normal as a result of the feeding, the nurse should notify the physician and anticipate that the doctor will order an intravenous of dextrose and water.

A baby has been admitted to the neonatal intensive care unit with a diagnosis of postmaturity. The nurse expects to find which of the following during the initial newborn assessment? 1. Abundant lanugo. 2. Flat breast tissue. 3. Prominent clitoris. 4. Wrinkled skin.

4 The postterm baby does have dry, wrinkled, and often desquamating skin. The baby's dehydration is secondary to a placenta that progressively deteriorates after 40 weeks' gestation.

A neonate is under phototherapy for elevated bilirubin levels. The baby's stools are now loose and green. Which of the following actions should the nurse take at this time? 1. Discontinue the phototherapy. 2. Notify the health care practitioner. 3. Take the baby's temperature. 4. Assess the baby's skin integrity.

4 The stools can be very caustic to the baby's delicate skin. The nurse should cleanse the area well and inspect the skin for any sign that the skin is breaking down.

While working with the pregnant woman in her first trimester, the nurse is aware that chorionic villus sampling (CVS) can be performed during pregnancy at: a. 4 weeks b. 8 weeks c. 10 weeks d. 14 weeks

c. 10 weeks

A baby presents with a HR of 180, decreased muscle tone, RR of 72, and the use of intercostal muscles when breathing. What does the nurse anticipate to educate the mother about? a. intraventricular hemorrhage b. respiratory distress syndrome c. facial paralysis d. sepsis

b. respiratory distress syndrome

In the first trimester, ultrasonography can be used to gain information on: a. Amniotic fluid volume. b. the presence of maternal abnormalities c. Placental location and maturity. d. Cervical length.

b. the presence of maternal abnormalities

A client, 34 weeks pregnant, arrives at the ER with SEVERE abdominal pain, uterine tenderness and an increased uterine tone. The client denies vaginal bleeding. The external fetal monitor shows fetal distress with severe, variable decels. The client most likely has which of the following? 1. Abruptio Plantae 2. Placenta Previa 3. Molar Pregnancy 4. Ectopic Pregnancy

1. Abruptio placentae a client w/ severe abruptio placentae will often have SEVERE abdominal pain. The uterus will have increased tone w/ little to no return to resting tone btw/ contractions. The fetus will start to show signs of distress, with decels in the HR or even fetal death w/ large placental separation. Placenta previa usually involves PAINLESS vaginal bleeding w/out UCs. A molar preg. generally would be detected before 34 weeks gestation. An ecoptic preg. which usually occurs in the FALLOPIAN TUBES, would rupture well before 34 weeks gestation

Clients with gestational diabetes are usually managed by which of the following therapies? 1. diet 2. long acting insulin 3. oral hypoglycemic drugs 4. oral hypoglycemic drugs/insulin

1. diet oral hypoglycemics are contraindicated in preg. long acting insulin usually inst needed for blood glucose control in the client with GDM

An 18-hour-old baby is placed under the bili-lights with an elevated bilirubin level. Which of the following is an expected nursing action in these circumstances? 1. Give the baby oral rehydration therapy after all feedings. 2. Rotate the baby from side to back to side to front every two hours. 3. Apply restraints to keep the baby under the light source. 4. Administer intravenous fluids via pump per doctor orders.

2

A nurse makes the following observations when admitting a full-term, breastfeeding baby into the neonatal nursery: 9 lbs 2 oz, 21 inches long, TPR: 96.6°F, 158, 62, jittery, pink body with bluish hands and feet, crying. Which of the following actions should the nurse perform first? 1. Swaddle the baby to provide warmth. 2. Assess the glucose level of the baby. 3. Take the baby to the mother for feeding. 4. Administer the neonatal medications.

2 The glucose level should be assessed to determine whether or not this baby is hypoglycemic. (1. This baby is hypothermic, but the best intervention would be to place the baby under a warmer rather than to swaddle the baby. Plus, the baby's glucose levels must be assessed in order to determine whether or not this baby is hypoglycemic. The glucose can be evaluated while the baby is under the warmer. 3. A feeding will elevate the glucose level if it is below normal. The nurse does need to assess the level, however, in order to make a clear determination of the problem. 4. The administration of the neonatal medicines is not a priority at this time.)

A client is seeking preconception counseling. She has type 1 diabetes mellitus and is found to have an elevated glycosylated hemoglobin (HgbA1c) level. Before actively trying to become pregnant, she is strongly encouraged to stabilize her blood glucose to reduce the possibility of her baby developing which of the following? 1. Port wine stain. 2. Cardiac defect. 3. Hip dysplasia. 4. Intussusception.

2 The incidence of cardiac defects and neural tube defects is high in infants born to diabetic mothers.

A 17 y.o. primpigravida with severe gHTN has been receiving mag sulfate IV for 3 hours. The latest assessment reveals DTR of +1, BP 150/100 mmgHg, pulse 92 bpm, respiratory rate 10bpm and urine output 20ml/hr. Which of the following actions would be most appropriate? 1. Continue monitoring per standards of care 2. Stop the mag sulfate infusion 3. Increase infusion by 5gtt/min 4. Decrease infusion by 5gtt/min

2. Stop the mag sulfate infusion Mag sulfate should be withheld if the clients resp rate or urine output falls or if reflexes are diminished / absent.

There is a baby in the neonatal intensive care unit (NICU) who is exhibiting signs of neonatal abstinence syndrome. Which of the following medications is contraindicated for this neonate? 1. Morphine. 2. Opium. 3. Narcan. 4. Phenobarbital.

3. Narcan.

The physician suspects that the client may have gestational trophoblastic disease. Which clinical manifestations support this diagnosis? (SATA) a. Increased levels of beta-hCG in the serum b. Fundal height correlating with reported gestational age c. Vaginal bleeding d. Vomiting e. Maternal hypotension

A, C, D ~ In gestational trophoblastic disease (molar pregnancy), the following clinical manifestations would appear: increased serum beta-hCG levels, increased size of the uterus related to gestational age, nausea and vomiting, and evidence of vaginal bleeding. Development of preeclampsia earlier in the pregnancy would be noted, resulting in hypertension, not hypotension.

The nurse is performing an admission assessment on a newborn infant with the diagnosis of subdural hematoma after a difficult vaginal delivery. Which assessment technique would assist to support the newborn's diagnosis? 1. Monitoring the urine for blood 2. Monitoring the urinary output pattern 3. Testing for contractures of the extremities 4. Stimulating for reflex responses in the extremities

4 A subdural hematoma can cause pressure on a specific area of the cerebral tissue. Especially if the infant is actively bleeding, such pressure can cause changes in the stimuli responses in the extremities on the opposite side of the body. Options 1 and 2 are incorrect. An infant after delivery normally would be incontinent of urine. Blood in the urine would indicate abdominal trauma and would not be a result of the hematoma. Option 3 is incorrect because contractures would not occur this soon after delivery.

Neonates born to women infected with hepatitis B should undergo which Tx regimen 1. Hep B vaccine at birth and 1 month 2. Hep B immune globulin at birth, no hepatitis B vaccine 3. Hepatits B immune globulin within 48 hours of birth and Hep B vaccine at 1 month 4. Hep B immune globulin within 12 hours of birth and Hep B vaccine at birth, 1 month, 6 months

4. Hep B immune globulin within 12 hours of birth and Hep B vaccine at birth, 1 month, 6 months Hep B immune globulin should be given as soon as possible after birth but within 12 hours

The nurse is monitoring a client in preterm labor who is receiving intravenous magnesium sulfate. The nurse should monitor for which adverse effects of this medication? Select all that apply. A. Flushing B. Hypertension C. Increased urine output D. Depressed respirations E. Extreme muscle weakness F. Hyperactive deep tendon reflexes

A, D, & E

With regard to congenital anomalies of the cardiovascular and respiratory systems, nurses should be aware that: A. Cardiac disease may be manifested by respiratory signs and symptoms. B. Screening for congenital anomalies of the respiratory system need only be done for infants having respiratory distress. C. Choanal atresia can be corrected by a suction catheter. D. Congenital diaphragmatic hernias are diagnosed and treated after birth.

A A. Correct: The cardiac and respiratory systems function together.B. Incorrect: Screening for congenital respiratory system anomalies is necessary even for infants who appear normal at birth.C. Incorrect: Choanal atresia requires emergency surgery.D. Incorrect: Congenital diaphragmatic hernias are discovered prenatally on ultrasound.

During a prenatal examination, the woman reports having two cats at home. The nurse informs her that she should not be cleaning the litter box while she is pregnant. When the woman questions the nurse as to why, the nurse's best response would be: A. "Your cats could be carrying toxoplasmosis. This is a zoonotic parasite that can infect you and have severe effects on your unborn child." B. "You and your baby can be exposed to the HIV virus in your cats' feces." C. "It's just gross. You should make your husband clean the litter boxes." D. "Cat feces are known to carry E. coli, which can cause a severe infection in both you and your baby."

A A. Correct: Toxoplasmosis is a multisystem disease caused by the protozoal Toxoplasma gondii parasite, commonly found in cats, dogs, pigs, sheep, and cattle. About 30% of women who contract toxoplasmosis during gestation transmit the disease to their offspring. Clinical features ascribed to toxoplasmosis include hydrocephalus or microcephaly, chorioretinitis, seizures, or cerebral calcifications. B. Incorrect: HIV is not transmitted by cats. C. Incorrect: Although this may be a valid statement, it is not appropriate, does not answer the client's question, and is not the nurse's best response. D. Incorrect: E. coli is found in normal human fecal flora. It is not transmitted by cats.

As the prenatal nurse analyzes the lab results of multiple pregnant clients seen that day for prenatal​ checks, which lab result would indicate that a client has​ preeclampsia? A. Elevated liver enzymes B. Increased platelet count C. Decreased creatinine D. Decreased urine​ protein/creatinine ratio

A ​Rationale: Elevated liver enzymes and​ decreased, not​ increased, platelet count would be of the greatest concern as it may be reflective of HELLP syndrome. This syndrome is also characterized by hemolysis. A decreased creatinine or urine​ protein/creatinine ratio is not associated with the pathology of preeclampsia.

The postpartum nurse is caring for a client who delivered a healthy but premature infant after induction at 35 weeks of gestation due to severe preeclampsia. Which intervention should the nurse implement postdelivery to monitor for improvement in the​ preeclampsia? A. Strict monitoring of intake and output​ (I &​ O) B. Assessing for excessive vaginal bleeding C. Weighing the client daily to assess for weight gain D. Monitoring blood pressure every hour for the first 48 hours

A ​Rationale: Strict monitoring of intake and output​ (I &​ O) is​ important, as diuresis should occur with the return of normal kidney​ function, indicating reversal of the disease process. Assessing for excessive vaginal bleeding and daily weight gain would not provide information concerning improvement of preeclampsia. The blood pressure should be monitored every 4 hours for 48​ hours, or every hour for 24 hours if on magnesium sulfate for severe preeclampsia.

A female client presents to the emergency room complaining of lower abdominal cramping with scant bleeding of approximately 2 days duration. This morning, the quality and location of the pain changed and she is now experiencing pain in her shoulder. The clients last menstrual period was 28 days ago, but she reports that her cycle is variable, ranging from 21 to 45 days. Which clinical diagnosis does the nurse suspect? a. Ectopic pregnancy b. Appendicitis c. Food poisoning d. Gastroenteritis

A ~ Even though the client's menstrual cycle has variability, all women are considered to be pregnant until proven otherwise. The client's presenting symptoms are typical for ectopic pregnancy, so the client should be monitored for the possible complication of rupture and shock.

The nurse is creating a plan of care for a pregnant client with a diagnosis of severe preeclampsia. Which nursing actions should be included in the care plan for this client? Select all that apply. A. Keep the room semi-dark. B. Initiate seizure precautions. C. Pad the side rails of the bed. D. Avoid environmental stimulation. E. Allow out-of-bed activity as tolerated.

A, B, C, & D

Which assessment is not included in the fetal biophysical profile (BPP)? a. Fetal movement b. Fetal tone c. Fetal heart rate d. Amniotic fluid index e. Placental grade

ANS: E Fetal movement, tone, heart rate, and amniotic fluid index are all assessed in a BPP. The placental grade is determined by ultrasound (as is a BPP), but it is not included in the criteria of assessment factors for a BPP.

A woman with gestational hypertension experiences a seizure. Which of the following would be the priority? A) Fluid replacement B) Oxygenation C) Control of hypertension D) Delivery of the fetus

Ans: B As with any seizure, the priority is to clear the airway and maintain adequate oxygenation both to the mother and the fetus. Fluids and control of hypertension are addressed once the airway and oxygenation are maintained. Delivery of fetus is determined once the seizures are controlled and the woman is stable.

When preparing a schedule of follow-up visits for a pregnant woman with chronic hypertension, which of the following would be most appropriate? A) Monthly visits until 32 weeks, then bi-monthly visits B) Bi-monthly visits until 28 weeks, then weekly visits C) Monthly visits until 20 weeks, then bi-monthly visits D) Bi-monthly visits until 36 weeks, then weekly

Ans: B For the woman with chronic hypertension, antepartum visits typically occur every 2 weeks until 28 weeks' gestation and then weekly to allow for frequent maternal and fetal surveillance.

Which assessment finding on the fetal monitor strip supports a diagnosis of abruptio placentae? a. FHR of 150 bpm b. Moderate variability of FHR c. Contractions every 3 minutes d. Uterine resting tone of 30 mm Hg

D ~ Abruptio placentae results in uterine irritability and a high resting uterine tone. A normal resting tone is from 5 to 15 mm Hg; FHR of 150 bpm, moderate variability of FHR, and contractions every 3 minutes are normal labor findings.

A woman with placenta previa is being treated with expectant management. The woman and fetus are stable. The nurse is assessing the woman for possible discharge home. Which statement by the woman would suggest to the nurse that home care might be inappropriate? A) "My mother lives next door and can drive me here if necessary." B) "I have a toddler and preschooler at home who need my attention." C) "I know to call my health care provider right away if I start to bleed again." D) "I realize the importance of following the instructions for my care."

Ans: B Having a toddler and preschooler at home needing attention suggest that the woman would have difficulty maintaining bed rest at home. Therefore, expectant management at home may not be appropriate. Expectant management is appropriate if the mother and fetus are both stable, there is no active bleeding, the client has readily available access to reliable transportation, and can comprehend instructions.

The nurse is developing a plan of care for a woman who is pregnant with twins. The nurse includes interventions focusing on which of the following because of the woman's increased risk? A) Oligohydramnios B) Preeclampsia C) Post-term labor D) Chorioamnionitis

Ans: B Women with multiple gestations are at high risk for preeclampsia, preterm labor, hydramnios, hyperemesis gravidarum, anemia, and antepartal hemorrhage. There is no association between multiple gestations and the development of chorioamnionitis.

A pregnant client has a blood pressure​ (BP) reading of​ 142/90 mmHg at the​ 32-week prenatal visit. Upon return 1 week​ later, the​ client's BP is​ 152/94 mmHg. Prior to these​ results, the client had normal BP readings. The prenatal nurse anticipates that this client will be diagnosed with which hypertensive disorder of​ pregnancy? A. Eclampsia B. Gestational hypertension C. Preeclampsia D. Chronic hypertension

B ​Rationale: Gestational hypertension occurs in the second half of pregnancy in a previously normotensive mother. Diagnosis is made after obtaining a BP greater than or equal to​ 140/90 mmHg on at least two occasions​ (at least 6 hours​ apart, after 20 weeks of​ gestation). Eclampsia is preeclampsia with the presence of seizures. Preeclampsia is defined according to the same criteria as gestational hypertension but is also accompanied by signs of end organ damage. Chronic hypertension is identified if it occurs in a pregnant client with a known history of hypertension prior to​ pregnancy, is discovered during the pregnancy prior to 20 weeks of​ gestation, or persists for more than 12 weeks postpartum

A pregnant client is receiving magnesium sulfate for the management of preeclampsia. The nurse determines that the client is experiencing toxicity from the medication if which findings are noted on assessment? Select all that apply. A. Proteinuria of 3+ B. Urine output of 20 mL in an hour C. Presence of deep tendon reflexes D. Respirations of 10 breaths/minute E. Serum magnesium level of 4 mEq/L (2 mmol/L)

B & D

Which information should the labor nurse recognize as being pertinent to a possible diagnosis of abruptio placentae? a. Low back pain b. Firm, tender uterus c. Regular uterine contractions d. Scant vaginal mucus drainage

B ~ A firm, tender uterus is a classic sign of abruptio placentae; low back pain, regular uterine contractions, and scant vaginal mucus drainage are normal findings in a laboring client.

Which assessment finding indicates an adverse response to magnesium sulfate? a. Urine output of 30 mL/hr b. Respiratory rate of 11 breaths/min c. Hypoactive patellar reflex d. Blood pressure reading of 110/80 mm Hg

B ~ A respiratory rate less than 12 breaths/min indicates magnesium toxicity and requires immediate intervention. A urine output of 30 mL/hr is normal urinary output; a hypoactive patellar reflex and blood pressure reading of 110/80 mm Hg are normal findings in the client receiving magnesium sulfate.

Which of these interventions should the nurse recognize as the priority for the client diagnosed with an intact tubal pregnancy? a. Assessment of pain level b. Administration of methotrexate c. Administration of Rh immune globulin d. Explanation of the common side effects of the treatment plan

B ~ The goal of medical management of an intact tube is to preserve the tube and improve the chance of future fertility. Methotrexate (a folic acid antagonist) is used to inhibit cell division and stop growth of the embryo. Assessment of pain level, administration of Rh immune globulin, and explaining common side effects of the treatment plan should be implemented in conjunction with or soon after treatment with methotrexate has begun.

A primigravida has just delivered a healthy infant girl. The nurse is about to administer erythromycin ointment in the infant's eyes when the mother asks, "What is that medicine for?" The nurse responds: A. "It is an eye ointment to help your baby see you better." B. "It is to protect your baby from contracting herpes from your vaginal tract." C. "Erythromycin is given prophylactically to prevent a gonorrheal infection." D. "This medicine will protect your baby's eyes from drying out over the next few days."

C A. Incorrect: Erythromycin has no bearing on enhancing vision. B. Incorrect: Erythromycin is used to prevent an infection caused by gonorrhea, not herpes. C. Correct: With the prophylactic use of erythromycin, the incidence of gonococcal conjunctivitis has declined to less than 0.5%. Eye prophylaxis is administered at or shortly after birth to prevent ophthalmia neonatorum. D. Incorrect: Erythromycin is given to prevent infection, not for lubrication.

Infants of mothers with diabetes are at higher risk for developing: A. Anemia B. Hyponatremia C. Respiratory distress syndrome D. Sepsis

C A. Incorrect: Infants of diabetic mothers (IDMs) are not at risk for anemia. They are at risk for polycythemia. B. Incorrect: IDMs are not at risk for hyponatremia. They are at risk for hypocalcemia and hypomagnesemia. C. Correct: IDMs are at risk for macrosomia, birth trauma, perinatal asphyxia, respiratory distress syndrome, hypoglycemia, hypocalcemia, hypomagnesemia, cardiomyopathy, hyperbilirubinemia, and polycythemia. D. Incorrect: IDMs are not at risk for sepsis.

The nurse has discussed methods to decrease the risk of seizure activity with a client diagnosed with preeclampsia. Which client statement indicates an understanding of the​ teaching? A. ​"I will keep my legs​ elevated." B. "I will let you know if I do not feel my baby​ move." C. "I will make sure everything is​ quiet." D. ​"I will not lay flat on my​ back."

C ​Rationale: Decreasing environmental stimuli helps reduce the risk of seizures that may occur for a client with preeclampsia. Lying on the left side maximizes uterine and renal profusion. Elevating the lower extremities helps prevent edema. Decreased fetal movement is associated with fetal hypoxia

You are taking care of a client who had a therapeutic abortion following an episode of vaginal bleeding and ultrasound confirmation of a blighted ovum. Lab work is ordered 2 weeks postprocedure as a follow-up to medical care. Which result indicates that additional intervention is needed? a. Hemoglobin, 13.2 mg/dL b. White blood cell count, 10,000 mm3 c. Beta-hCG detected in serum d. Fasting blood glucose level, 80 mg/dL

C ~ The presence of beta-hCG in serum 2 weeks after the procedure is clinically significant and indicates the possibility that there may have been a molar pregnancy (hydatidiform). Thus, further examination is required. None of the other lab results warrant intervention because they are within normal limits.

Compared with contraction stress test (CST), nonstress test (NST) for antepartum fetal assessment: a. Has no known contraindications. b. Has fewer false-positive results. c. Is more sensitive in detecting fetal compromise. d. Is slightly more expensive.

a. Has no known contraindications.

The nurse is caring for a preterm infant with necrotizing enterocolitis (NEC). Which nursing intervention is most important for this infant? a. Diluting the formula mixture as prescribed b. Measuring abdominal girth frequently c. Using half-strength formula for tube feeding d. Administering oxygen before the gastric feeding

b. Measuring abdominal girth frequently NEC is marked by prolonged gastric emptying; an increase in abdominal girth of more than 1 cm in 4 hours is significant and requires immediate intervention. Formula feeding is stopped and parenteral fluids, usually total parenteral nutrition (TPN), are started instead. Administering oxygen before the gastric feeding will have no therapeutic value for an infant with NEC.

If a 12 week pregnant patient has spotting with a closed cervix and mild cramping then this is what type of miscarriage? a. inevitable miscarriage b. incomplete miscarriage c. threatened miscarriage d. complete miscarriage

c. threatened miscarriage

In comparing the abdominal and transvaginal methods of ultrasound examination, nurses should explain to their clients that: a. Both require the woman to have a full bladder. b. The abdominal examination is more useful in the first trimester. c. Initially the transvaginal examination can be painful. d. The transvaginal examination allows pelvic anatomy to be evaluated in greater detail.

d. The transvaginal examination allows pelvic anatomy to be evaluated in greater detail.

A macrosomic baby in the nursery is suspected of having a fractured clavicle from a traumatic delivery. Which of the following signs/symptoms would the nurse expect to see? (Select all that apply) 1. Pain with movement. 2. Hard lump at the fracture site. 3. Malpositioning of the arm. 4. Asymmetrical Moro reflex. 5. Marked localized ecchymosis

1, 2, 3, 4 1. The baby will complain of pain at the site. 2. If not in the immediate period after the injury, within a few days there will be a palpable lump on the bone at the site of the break. 3. Because of the break, the baby is likely to position the arm in an atypical posture. 4. Because of the injury to the bone, the baby is unable to respond with symmetrical arm movements. (5. It is very rare to see ecchymosis at the site of the break.)

A newborn in the nursery is exhibiting signs of neonatal abstinence syndrome. Which of the following signs/symptoms is the nurse observing? (Select all that apply) 1. Seizures. 2. Lethargy. 3. Tremors. 4. Hyporeflexia. 5. Persistent shrill cry

1, 3, & 5 are correct. I changed it so rational is gonna be different 1. Babies with signs of neonatal abstinence syndrome repeatedly exhibit signs of hunger. 2. Babies with neonatal abstinence syndrome are hyperactive, not lethargic. drome often exhibit sleep disturbances rather than prolonged periods of sleep. 4. Babies with signs of neonatal abstinence syndrome are hyperreflexic, not hyporeflexic. 5. Babies with signs of neonatal abstinence syndrome often have a shrill cry that may continue for prolonged periods.

A client who is 32 weeks pregnant is being monitored in the antepartum unit for gHTN. She suddenly complains of continuous abdominal pain and vaginal bleeding. Which of the following nursing internventions should be included in the care of this client? Check all that apply 1. Evaluate VS 2. Prepare for vaginal delivery 3. Reassure client that she'll be able to continue pregnancy 4. Evaluate FHT 5. Monitor amt of vaginal bleed 6. Monitor I&O

1, 4, 5, 6 The clients symptoms indicate that she's experiencing abruptio placenta. The nurse must immed eval the moms well being by eval VS, FWB, by auscultation of heart tones, monitoring amt of blood loss and eval the vol status by measuring I&O. After the severity of the abruption has been determined and blood and fluid have been replaced, prompt C-SECTION delivery of the fetus (not vaginal) is indicated if the fetus is in distress

Which of the following is best to monitor a fetus of a client with diabetes in her 3rd trimester 1. US exam weekly 2. NST 2x/week 3. Daily contraction stress test at 32 weeks 4. monitoring fetal activity by client weekly

2. NST 2X/week NST is the preferred antepartum HR screening test for pregnant clients with diabetes. NSTs should be done at least 2x per week staring at 32 weeks gestation, as fetal deaths in clients w/ diabetes have been noted within 1 week of a reactive NST. US should be done ever 4-6 weeks to monitor fetal growth. CST wouldnt be initiated at 32 weeks Maternal fetal activity monitoring should be done daily

The nurse is developing a plan of care for a preterm newborn infant. The nurse develops measures to provide skin care, knowing that the preterm newborn infant's skin appears in what way? 1. Thin and gelatinous, with increased subcutaneous fat 2. Thin and gelatinous, with increased amounts of brown fat 3. Reddened, translucent, and gelatinous, with decreased amounts of subcutaneous fat 4. With fine downy hair on thin epidermal and dermal layers, with increased amount of brown fat

3 The skin of a newborn infant plays a significant role in thermoregulation and as a barrier against infection. The skin of a preterm newborn infant is immature in comparison with that of a term newborn infant. The skin of a preterm newborn is thin and gelatinous, with decreased amounts of subcutaneous fat, brown fat, and glycogen stores. In addition, preterm newborn infants lose heat because of their large body surface area in relation to their weight and because their posture is more relaxed, with less flexion. Therefore preterm newborn infants are less able to generate heat, which places them at risk for increased heat loss and increased fluid requirements

Which newborn is most at risk for a brachial plexus injury? 1. A term infant with a history of a forceps-assisted delivery 2. A term infant delivered via primary cesarean section for malpresentation 3. A large for gestational age infant with a history of shoulder dystocia at delivery 4. A 36-week preterm infant delivered vaginally after preterm rupture of membranes

3 Brachial plexus injuries, a fractured clavicle, or a fractured humerus are all possible risks during a delivery of an infant with shoulder dystocia and must be considered during the immediate newborn assessment. Stretching or pulling away of the shoulder from the head may occur during a difficult delivery such as one involving shoulder dystocia. This positioning may cause damage to the upper plexus. Larger infants are more likely to be involved in a delivery in which inadequate space is a concern. In most cases, option 4 would result in an infant of smaller size, so shoulder dystocia would not be a priority risk. Shoulder dystocia does not occur during cesarean section, which eliminates option 2. Option 1 can be eliminated because with a forceps delivery, priority concern is for facial or head injuries more than shoulder, arm, or clavicle injuries.

A neonate whose mother is HIV positive is admitted to the NICU. A nursing diagnosis: Risk for infection related to perinatal exposure to HIV/AIDS is made. Which of the following interventions should the nurse make in relation to the diagnosis? 1. Monitor daily viral load laboratory reports. 2. Check the baby's viral antibody status. 3. Obtain an order for antiviral medication. 4. Place the baby on strict precautions.

3 The standard of care for neonates born to mothers with HIV/AIDS is to begin them on anti-AIDS medication in the nursery. The mother will be advised to continue to give the baby the medication after discharge. (1. The baby will have a positive antibody titer, as a result of passive immunity through the placenta, but there will be no evidence of active viral production that early in the newborn's life. 2. There is no need to assess the antibody titer. It will definitely be positive because the mother has HIV/AIDS. 4. There is no need to place the baby on strict precautions. The institution of standard precautions in the well-baby nursery is sufficient.)

A baby has just been admitted into the neonatal intensive care unit with a diagnosis of intrauterine growth restriction (IUGR). Which of the following maternal factors would predispose the baby to this diagnosis? Select all that apply. 1. Hyperopia. 2. Gestational diabetes. 3. Substance abuse. 4. Chronic hypertension. 5. Advanced maternal age.

3, 4, and 5 are correct. 1. Hyperopia, another name for farsightedness, is unrelated to placental function. 2. If the mother had gestational diabetes, the nurse would expect the baby to be macrosomic, not to have IUGR. 3. Placental function is affected by the vasoconstrictive properties of many illicit drugs, as well as by cigarette smoke. 4. Placental function is diminished in women who have chronic hypertension. 5. Placental function has been found to be diminished in women of advanced maternal age.

Which TORCH infection could be contracted by the infant because the mother owned a cat? A. Toxoplasmosis B. Varicella zoster (chicken pox) C. Parvovirus B19 D. Rubella

A A. Correct: Cats that eat birds infected with the Toxoplasma gondii protozoan excrete infective oocysts. Humans (including pregnant women) can become infected if they fail to wash their hands after cleaning the litter box. The infection is passed through the placenta. B. Incorrect: Cats that eat birds infected with the Toxoplasma gondii protozoan excrete infective oocysts. Humans (including pregnant women) can become infected if they fail to wash their hands after cleaning the litter box. The infection is passed through the placenta. C. Incorrect: Cats that eat birds infected with the Toxoplasma gondii protozoan excrete infective oocysts. Humans (including pregnant women) can become infected if they fail to wash their hands after cleaning the litter box. The infection is passed through the placenta. D. Incorrect: Cats that eat birds infected with the Toxoplasma gondii protozoan excrete infective oocysts. Humans (including pregnant women) can become infected if they fail to wash their hands after cleaning the litter box. The infection is passed through the placenta.

The emergency room charge nurse calls the labor and birth charge nurse and reports the ambulance is en route with a seizing pregnant patient at 36 weeks gestation. What medication will the charge nurse most likely direct the staff nurse to prepare to administer immediately on the patients arrival to the labor and birth unit? a. Magnesium sulfate (magnesium) b. Hydralazine (Apresoline) c. Carbamazepine (Tegretol) d. Terbutaline (Brethine)

A ~ Magnesium sulfate is the drug most often used for preeclamptic and eclamptic patients. It is a CNS depressant. Apresoline is administered for hypertension and is often given to pregnant clients with severe preeclampsia. Tegretol is administered for seizure activity in nonpregnant patients. Brethine is a smooth muscle relaxant administered for preterm labor.

A 32-year-old primigravida is admitted with a diagnosis of ectopic pregnancy. Nursing care is based on which of the following? a. Hemorrhage is the major concern. b. She will be unable to conceive in the future. c. Bed rest and analgesics are the recommended treatment. d. A D&C will be performed to remove the products of conception.

A ~ Severe bleeding occurs if the fallopian tube ruptures. If the tube must be removed, her fertility will decrease but she will not be infertile. The recommended treatment is to remove the pregnancy before hemorrhaging. A D&C is done on the inside of the uterine cavity. The ectopic is located within the tubes.

For clinical purposes, the most accurate definition of preterm and postterm infants is defined as what? a. Preterm: Before 34 weeks of gestation if the infant is appropriate for gestational age (AGA); before 37 weeks if the infant is small for gestational age (SGA) b. Postterm: After 40 weeks of gestation if the infant is large for gestational age (LGA); beyond 42 weeks if the infant is AGA c. Preterm: Before 37 weeks of gestation and postterm beyond 42 weeks of gestation; no matter the size for gestational age at birth d. Preterm: Before 38 to 40 weeks of gestation if the infant is SGA; postterm, beyond 40 to 42 weeks gestation if the infant is LGA

ANS: C Preterm and postterm are strictly measures of time—before 37 weeks and beyond 42 weeks, respectively—regardless of the size for gestational age.

The most common cause of pathologic hyperbilirubinemia is: A. Hepatic disease B. Hemolytic disorders in the newborn C. Postmaturity D. Congenital heart defect

B A. Incorrect: Hepatic damage may be a cause of pathologic hyperbilirubinemia, but it is not the most common cause. B. Correct: Hemolytic disorders in the newborn are the most common cause of pathologic jaundice. C. Incorrect: Prematurity would be a potential cause of pathologic hyperbilirubinemia in neonates, but it is not the most common cause. D. Incorrect: Congenital heart defect is not a common cause of pathologic hyperbilirubinemia in neonates.

A home care nurse is monitoring a 16-year-old primigravida who is at 36 weeks' gestation and has gestational hypertension. Her blood pressure during the past 3 weeks has been averaging 130/90 mm Hg. She has had some swelling in the lower extremities and has had mild proteinuria. Which statement by the woman should alert the nurse to the worsening of gestational hypertension? A. "My vision for the past 2 days has been really fuzzy." B. "The swelling in my hands and ankles has gone down." C. "I had heartburn yesterday after I ate some spicy foods." D. "I had a headache yesterday, but I took some acetaminophen and it went away."

a

Nurses should be aware that the biophysical profile (BPP): a. Is an accurate indicator of impending fetal well being. b. Is a compilation of health risk factors of the mother during the later stages of pregnancy. c. Consists of a Doppler blood flow analysis and an amniotic fluid index. d. Involves an invasive form of ultrasound examination.

a. Is an accurate indicator of impending fetal well being.

A 37 week neonate is born and displays signs of fetal alcohol syndrome. What are initial findings that the nurse recognizes related to this? Select all that apply a. flat face shape b. macrocephaly c. thin upper lip d. bulging eyes e. large nose

a. flat face shape c. thin upper lip

A herpes simplex virus (HSV) positive mother questions the nurse about if breastfeeding is safe or not. What is the best response from the nurse? a. Yes, breastfeeding is always safe b. Yes, but only if no lesions are present on the breast c. Yes, but only with a nipple shield d. No, breastfeeding is not safe with HSV

b. Yes, but only if no lesions are present on the breast

The nurse providing care for the antepartum woman should understand that contraction stress test (CST): a. Sometimes uses vibroacoustic stimulation. b. Is an invasive test; however, contractions are stimulated. c. Is considered negative if no late decelerations are observed with the contractions. d. Is more effective than nonstress test (NST) if the membranes have already been ruptured.

c. Is considered negative if no late decelerations are observed with the contractions.


संबंधित स्टडी सेट्स

Group Sickness and Accident Insurance

View Set

Agency Law - Introduction, Types, Duties

View Set

contract law end of chapter quiz

View Set

Abeka Science: Order & Design ( 7th Grade) Test 9 Chapters 7-9

View Set

Week 4 Qualitative Research (Part 1)

View Set

Medication Administration Assessment

View Set

Genética. El genoma humano. Capítulo 2

View Set